You are on page 1of 60

JEE (MAIN+ADVANCED)

PERMUTATION & COMBINATION


CONTENT

S.No Pages

1. Theory 01 – 28

2. Exercise-1 (Special DPP) 29 – 43

3. Exercise-2 44 – 46

4. Exercise-3 (Section-A) 47 – 49
[Previous years JEE-Advanced problems]

5. Exercise-3 (Section-B) 49 – 50
[Previous years JEE-Main problems]

6. Exercise-4 (Potential Problems for Board Preparations) 51

7. Exercise-5 (Rank Booster) 52

8. Answer Key 53 – 55
PERMUTATION AND COMBINATION

PERMUTATION AND COMBINATION

FUNDAMENTAL PRINCIPAL OF COUNTING :

If an event can occur in m different ways following which another event can occur in n different ways,
then total number of simultaneous occurrence of both the events in a definite order is (m × n). This can
be extended to any number of events.
Eg : For an event to be occur in m1, m2, ……, mn ways then number of ways is m1 × m2 …… × mn

Important Point :
FPC (Fundamental Principal of Counting) is used to count some event without actually counting
them.

Let us take help of some model.

Model- I : Find number of ways of in which one can travel from T1 (town 1) to T3 (town 3) via T2 (town 2).

R1 R4
R2
T1 T2 T3
R3
R5
Total ways : –

T1 R1 T2 R4 T3

T1 R1 T2 R5 T3

T1 R2 T2 R4 T3

T1 R2 T2 R5 T3

T1 R3 T2 R4 T3

T1 R3 T2 R5 T3 = 6 ways

It is easy to proceed by FPC T1 to T2  3


T2 to T3  2
Total ways = 3 × 2 = 6

BANSAL CLASSES Private Ltd. ‘Bansal Tower’, A-10, Road No.-1, I.P.I.A., Kota-05 Page # 1
PERMUTATION AND COMBINATION

Model- II :
(i) To find the number of ways by which a person can enter and leave cinema hall by a different door.

D2
D3
D1 D4
D5
Cinema Hall D1
D3
D2 D4
D1 D5 D5
D1
Cinema D2
D3 D4
Hall
D2 D4 D5
D1
D3 D4 D2 4 + 4 + 4 + 4 + 4 = 20
D3
D5
D1
D2
D5 D3
D4

By F.P.C.
A person can enter in cinema hall by 5 ways & leave by 4 ways = 5 × 4 = 20.

(ii) If he can enter and leave by any door then number of ways = 5 × 5 = 25.

Basic Steps to Remember :


Step-I : Identify the independent events involved in a given problem.
Step-II : Find the number of ways performing/occurring each event
Step-III : Multiplythese numbers to get the total number of ways of performing/occurring all the events

Illustration :
How many 3 digit numbers can be formed by the digit 1, 2, 3, 4, 5 without repetition.
Sol. Hundred's place digit can be selected in 5 ways.
Ten's place digit can be selected in 4 ways.
Unit's place digit can be selected in 3 ways.
So, 5 × 4 × 3 = 60

BANSAL CLASSES Private Ltd. ‘Bansal Tower’, A-10, Road No.-1, I.P.I.A., Kota-05 Page # 2
PERMUTATION AND COMBINATION

Lexicography illustration (Lexicography is called science of making words) :

(a) Find total number of 5 letter word that can be formed from letters of word "TOUGH".

5 × 4 × 3 × 2 × 1= 120

(b) Find the rank of "TOUGH" if all the letters of the word are arranged in all possible orders & written out
as in a dictionary.

The number of letters in the word "TOUGH" is 5 & all the five letters are different.
Alphabetical order of all the letters is G, H, O, T, U
Number of words beginning with G = 4×3×2×1
Number of words beginning with H = 4×3×2×1
Number of words beginning with O = 4×3×2×1
Number of words beginning with TG = 3×2×1
Number of words beginning with TH = 3×2×1
Number of words beginning with TOG = 2×1
Number of words beginning with TOH = 2×1
Next words beginning with "TOU" and it is "TOUGH" = 1.
Rank = 24 + 24 + 24 + 6 + 6 + 2 + 2 + 1 = 89

NOTATION OF FACTORIAL :
Notation of factorial & its Algebra :
The continued product of first n, natural number is called as " n factorial" and denoted by n! or n
n! = 1 · 2 · 3 · 4 · …… (n – 1)n!
4! = 4 · 3 · 2 · 1 = 24
3! = 1 · 2 · 3 = 6
5! = 120; 61 = 720; 7! = 5040

Special Results :
 0! = 1 i.e. factorial of zero is 1
Proof : n ! = (n – 1)! · n
Putting n = 1
1! = (1 – 1)! 1  0! = 1
 Factorial of negative number is undefined
n! 0! 1
(n – 1)! = if n = 0 then (–1)! = = Not defined.
n 0 0
Asking:
(i) Find n if (n + 1)! = 12 × (n – 1)!
(ii) (n + 2)! = 2550 n!
Sol.
(i) (n + 1) n (n – 1)! = 12 × (n – 1)!
n2 + n – 12 = 0; (n + 4) (n – 3) = 0; tn = 3

(ii) (n + 2) (n + 1) = 2550; (n + 52) (n – 49) = 0  n = 49

BANSAL CLASSES Private Ltd. ‘Bansal Tower’, A-10, Road No.-1, I.P.I.A., Kota-05 Page # 3
PERMUTATION AND COMBINATION

 (2n!) = 2n · n! [1 · 3 · 5 …… (2n – 1)]


Proof :
(2n)! = 1 · 2 · 3 · 4 · 5 · 6 …… (2n – 1) (2n – 2)
Take 2 common each n even terms
= 2n (1 · 3 · 5 …… (2n + 1)) (1 · 2 · 3 …… n)
= 2n (n!) (1 · 3 · 5 …… (2n + 1))

Basic Method :
Now we decide number of 5s in (1000)! (1000!)
5, 10, 15 ……… 1000 : 200  200 number containing at least one 5
25, 50 ……… 1000 : 40  40 number containing at least two 5
125, 250 ……… 1000 8  8 number containing at least three 5
625 1  1 number containing at least four 5
Total = 200 + 40 + 8 + 1 = 249
Objective approach
1000  1000  1000  1000 
E5 (1000!) =    
 5   5   5   5 
2 3 4

= 200 + 40 + 8 + 1 = 249
Where [.] denotes greatest integer function.

DIVISIBILITY OF NUMBERS :
The following chart shows the conditions of divisibility of numbers by 2,3,4,5,6,8,9,25
Divisible by Condition
2 whose last digit is even (2, 4, 6, 8, 0)
3 sum of whose digits is divisible by 3
4 whose last two digits number is divisible by 4
5 whose last digit is either 0 or 5
6 which is divisible by both 2 and 3
8 whose last three digits numbers divisible by 8
9 sum of whose digits is divisible by 9
25 whose last two digits are divisible by 25

PERMUTATION & COMBINATION :


Permutation :
Permutation means arrangement in a define order of things which maybe alike or different taken some or
all at a time. Hence permutation refers to the situation where order of occurrence of the events is important.
OR
Permutation is arrangement of things in same definite order

Combination :
Combination refers to the situation where order of occurrence of the event is not important.
or combination is selection of one or more things out of n things.

BANSAL CLASSES Private Ltd. ‘Bansal Tower’, A-10, Road No.-1, I.P.I.A., Kota-05 Page # 4
PERMUTATION AND COMBINATION

Note : Things which are alike and which are different.All god made things in general area treated to be different
and all man made things are to be spelled weather alike or different.
Hence we say that permutation is arrangement of things in definite order.

Example :
(i) Out of A, B, C, D take 3 letters & form number plate of car. [Pemutation]

(ii) Out of four letters A, B, C, D take any 3 letters & form triangle (possible). [Combination]
st nd
In 1 case arrangement of letters are there, in 2 case only selection will form the triangle, arrangement
is not required.

Theorem related to application of Permutation and Combination :

Theorem-1 :
Number of permutations of n distinct things taken all at a time symbolised as :
nP
n = P(n, n) = A nn = n!
Proof :
Let these are n things arranged at n places
n · (n – 1) · (n – 2) …… 3 · 2 · 1 = n!
We also saythat number of ways in whichn distinct objects can be arranged amongst themselves in nPn = n!
i.e. Find total number of words that of 10 letters that can be formed from all the letters of word
GANESHPURI.
A = nPn = 10! = n!

Theorem-2 :
Number of permutations of n distinct things taken r at a time
0rn
n!
nP = P (n, r) = A rn =
r (n  r )!
Things T1, T2 …… Tn
Places 1, 2, 3, …… r
Choice n · (n – 1) · (n – 2) …… [n – (r – 1)]
n (n  1)(n  2)(n  r  1)(n  r )! n!
Total way = 
(n  r )! (n  r )!
Hence we can say that

nP
n!
r =
(n  r )!
Eg : In how many ways can 5 person be made to occupy 5 chairs
(a) Five different chairs
5P = 5! = 120
5
(b) Three different chairs
5P = 5 × 4 × 3 = 60
3

BANSAL CLASSES Private Ltd. ‘Bansal Tower’, A-10, Road No.-1, I.P.I.A., Kota-05 Page # 5
PERMUTATION AND COMBINATION

Theorem-3 :
Number of combination / selection of n distinct things taken r at a time
n n!
nC = (n, r) =   
r  r  (r )!(n  r )!
Proof :
Let 10 different objects are given as A, B, C, D, E, F, G, H, I, J
Let combinations taking 3 at a time = x
Arrangement = (x) × (3!)
x · 3! = 10P3
10
P3 10!
x= =
3! (10  3)3!
Theorem-4 :
Number of combination of n different things taken r at a time when p perpendicular things are always
included. = n–pCr–p
i.e. Find total number of ways of selecting 11 player out of 15 player when Mahendra Sing Dhoni and
Yuvraj Singh are always included = 15 – 2 C11– 2  13C 9

Theorem-5 :
Number of combination of n different things taken r at a time when p particular thing are always excluded.
n–pC
r
Eg : How many different selections of 6 books can be made from 11 different books if two particular
books are never selected = 11– 2 C 6  9 C 6

Important Results :
(i) nC = 1
0
(ii) nC = 1
n
(iii) nC n
n–r = Cr  nC = nC
x y  x+y=n or x=y
(iv) n n
Pr = Cr r!
i.e. permutation is defined total number of combination of object then arrangement of objects.

Examples on nCr and nPr :

Illustration :
There are n points in a plane, no two of which collinear, find
(a) Number of straight lines
(b) Number of triangles
(c) Number of diagonals in a polygon of n sides.
Sol.
(a) Number of ways by which we can select any two points gives total number of straight lines = nC2.
(b) Ways by which can select any 3 non collinear points gives total number of triangle = nC3 .
(c) In any polygon number of vertices = number of sides
nC = number of sides + number of diagonals
2
nC – n = Number of diagonals.
2

BANSAL CLASSES Private Ltd. ‘Bansal Tower’, A-10, Road No.-1, I.P.I.A., Kota-05 Page # 6
PERMUTATION AND COMBINATION

Illustration :
There are 10 points in a plane of which 4 are collinear and rest are non-collinear. Find
(i) Number of lines (ii) Number of triangles
Sol.
(i) Number of lines
3 4
2 5
1 6
A 7 8 9 10 B
Method-I : Given 4 points are collinear so total number of ways of selecting any two points = 10C2
. If 4 collinear points give onlyone line (AB). So over counted number of lines formed by
collinear points = 4C2 – 1.
Thus total lines = 10C2 – 4C2 + 1
Method-II : Take any two points from upper arc = 6C2 ways.
Take one point from upper arc and one point from line AB = 6C1 × 4C1 ways.
Take both the points from lineAB then number of lines = 1 (LineAB).
Total number of lines = 6C1 × 4C1 + 6C2 + 1
(ii) Method-I : Number of ways of taking any three points = 10C3
Number of ways of taking any three points from four collinear points = 4C3
Number of triangles formed = 10C3 – 4C3
Method-II : Two points from upper arc and one point from line AB = 6C2 × 4C1
Two points from line AB and one point from upper arc = 6C1 × 4C2
All the three points from upper arc = 6C3
Total number of triangle = 6C2 × 4C1 + 6C1 × 4C2 + 6C3

Illustration :
In a morse telegraphy there are 4 arms and each arm is capable of taking 5 distinct position including the
position of rest. Find total no. of different signals.
1234R 1234R 1234R 1234R
Sol. One arm is capable of taking 5 positions
Hence total number of ways = 54
Out all above these arise a unique condition in arms
which no signal is transfer. It is R R R R
Hence number of different signal = 54 – 1

Illustration :
An 8 oared boat to be manned from a crew of 11 of which 3 can only steer. In how many ways the staff
can be arranged if 2 of the men can row only on the bow side.
Crew of 1
5 1
6 2
Stroke S Bow
3 can steer only 8 can only row 7 3
side side
8 4

2 only bow side 6 any where

BANSAL CLASSES Private Ltd. ‘Bansal Tower’, A-10, Road No.-1, I.P.I.A., Kota-05 Page # 7
PERMUTATION AND COMBINATION

Sol. For steering position 3 person's can be selected in 3C1 ways.


Out of 4 places of bow sides two are selected for 2 men by 4C2 ways and arranged by 4C2× 2! ways
Remaining six person's can be arranged by 6! ways at six places (As number of seats and number of
person's are equal)
3 4
Thus required ways = C1 × ( C2 (2!)) × (6!)
Row Any
Steer side side

Illustration :
4 Boys & 4 Girls are to be seated in a line find number of ways
(i) They can be seated so that "No two girls are together"
(ii) If not all the girls are together
or
If at least one girl is separated from rest of girls.
(iii) Boys and girls are alternate
(iv) If there are 4 married couples then number of ways in which they can be seated so the at each couple is
together.
Sol.
(i) Out of four standing boys five gaps are there
B1 B2 B3 B4
Out of five select any four gaps by 5C4 ways in which girls are arranged by = 5C4 × 4! ways.
Also standing boys are arranged by 4! ways
Required ways = (4!) × (5C4  4!)
Note :-Arrangement by this method is called as gap method.
(ii) Consider G1 G2 G3 G4  as one string.
Four boys
If all girls are together then total ways = (1 + 4)! (4!) = (5! × 4!)
Arrangement of four
String girls in string
Total number of arrangement without any restriction = 8!
Total number of ways by which not all girls are together = 8 ! – (5 ! × 4 !)
(iii) If boys and girls are alternate then two ways of arranging respective position of boys and girls are
shown below

OR

Number of ways of arranging boys  4!


Number of ways of arranging girls  4!
Required ways = 2 × 4! × 4!

BANSAL CLASSES Private Ltd. ‘Bansal Tower’, A-10, Road No.-1, I.P.I.A., Kota-05 Page # 8
PERMUTATION AND COMBINATION

(iv) Let the couples are as below


B1G 1 B 2G 2 B 3G 3 B 4G 4
There are four string and each string has husband and wife which can be arranged in 2! ways.
Ways of arranging strings = 4!
Required ways = (2!)4 × 4!
Illustration :
In how many ways 10 examination papers be arranged so that the best and the worst papers
never come together?
Sol. For the number of arrangements of 10 examination papers without restriction
n = 10 r = 10
total number of arrangement of 10 examination papers with any restriction = 10P10 = 10!
Keeping best and worst together, number of different papers = 9.
(i.e. in this case n = 9, r = 9)
Number of arrangements when best and worst are kept together = 9P9 · 2!
Hence, the required number of arrangements of 10 examination papers so that best and worst
papers are not together is 10! – 9P9 · 2! = 8 × 9!

FORMATION OF GROUPS :
(I) No. of ways in which (m + n) different things can be divided in two groups, one containing m things and
other contains 'n' things is
(m  n ) !
m+nC
n or m! n! .
Illustration :
Out of four players P1, P2, P3, P4 form two teams one contain 3 players and other one player.
4!
Sol. Number of groups = =4
1! 3!
Proof :
Divide P1, P2, P3, P4 in three groups
These an 4 ways

Select - 3 Rejected - 1
P1, P2 , P3 P4
P2 , P3 , P4 P1
P3 , P4 , P1 P2
P4 , P1, P2 P3

(II) If groups are equal size i.e. m = n


2n
Cn
Total number of ways in which 2n different things can be divided into two equal groups =
2!
( 2n ) !

(n!) (n!)(2!)
We divide by 2! to avoid false counting.

BANSAL CLASSES Private Ltd. ‘Bansal Tower’, A-10, Road No.-1, I.P.I.A., Kota-05 Page # 9
PERMUTATION AND COMBINATION

Proof :
Divide P1, P2, P3, P4 in two groups
Team A Team- B
P1P2 P3P4
P1P3 P2P4
P1P4 P2P3
P2P3 P1P4
P2P4 P1P3
P3P4 P1P2
We see that half of the case are repeated.
4!
Thus gives us wrong answer..
2! 2!
4!
Correct answer =
(2!)(2!)(2!)
Actually counting all such cases we observe that regrouping appear's when equal size group's
are required. To avoid false counting we divide by factorial of number of equal size group's.
(III) Total numbers of ways in which (m + n + p) different things can be divided into three unequal groups
( m  n  p) !
m, n, p is m! n! p!

Illustration :
If we divide 12 different things in three unequal gupas (2, 3, 7) then total number of ways are
2
12 diff. 12!
Sol. things 3
2! 3! 7!
4
Explanation :
2 2
12! 10!
12 12
2! 10! 3 3! 7!
10 10
7
12! 10! 12!
Total ways = 12C2 × 10C7 × 3C3 = × =
2! 10! 3! 7! 2! 3! 7!

If groups are equal (m = n = p) then number of ways


(3n )!
=
(n!)3 3!

BANSAL CLASSES Private Ltd. ‘Bansal Tower’, A-10, Road No.-1, I.P.I.A., Kota-05 Page # 10
PERMUTATION AND COMBINATION

Important Points :
The number of ways in which 'r' group's of n different object's can be formed in such a way that 'p'
groups of n1 object, q group of n2 object each is ……
n!
Required ways =
(n1!) (n 2!)q (p!)(q!)
p

n = (n1 + n1 ............. p times ) + (n2 + n2 ............ q times)


Divide byfactorial of number of equal size group.
Illustration :
In how many ways 3 team's of 11 player's each, 4 team's 6 player's each, 2 team's of 15 player's each
can be formed out of 87 player's
87 ! 1
Sol.  Required ways = 3 4 2

(11!) (6!) (15!) (3!)( 4!)( 2!)

Illustration :
Total number of ways in which 200 person's can be divided into 100 equal group's.
200!
Sol. Required ways = 100
(2!) (100)!
Corollary : Grouping and then arrangement.
If (m + n + p) different thing's can be divided in 3 group's & can be distributed to three person's.
(m  n  p)!
Required ways =  3!
m! n ! p!
Illustration :
In how many ways six different books can be distributed between four persons, so that each
person gets atleast one book.
Sol. Two cases possible {1, 1, 1, 3}, {1, 1, 2, 2}
 6! 6! 
Groups  3
 2 2  4!
 ( 1! ) 3! 3! ( 1! ) ( 2 ! ) 2! 2! 

BANSAL CLASSES Private Ltd. ‘Bansal Tower’, A-10, Road No.-1, I.P.I.A., Kota-05 Page # 11
PERMUTATION AND COMBINATION

PERMUTATION OF ALIKE OBJECTS :

Case  I : taken all at a time




Case  II : taken some at a time

Case-I: Permutation of alike objects taken all at a time


p of one kind  n!
Number of permutation of n things q of another kind and  taken all at a time =
r are all different  p!q!r!
Proof :
DAD DY
Let three D's are different
D1 A D2 D3 Y
One D A D D Y is counted as
D1 A D2 D3 Y six different, word's
D2 A D3 D1 Y
Let x is required ways
D2 A D1 D3 Y x × (6) = 6!
D3 A D2 D1 Y 6!
x=
D3 A D1 D2 Y 3!

Illustration :
Find total number of word's formed by using all letters of the word "IITJEE".
6!
Sol. ways are =
2! ( 2! )

Illustration :
Consider word ASSASSINATION, find number of ways of arranging the letters.
(i) Number of words using all.
(ii) If no two vowels are together.
(iii) If all S are seperated.
(iv) Atleast one S is seperated from rest of the S's
Sol.
(i) ASSASSINATION contains four S, three A, two N and two I.
13 !
Total ways =
( 4 ! )( 3 ! )( 2 ! )( 2 ! )

(ii) We have six vowels as A, A, A, I, I, O and seven consonants as S, S, S, S, N, T, N


|S|S|S|S|N|T|N|
Six vowels in 8 gap's
6! 7!
Total ways = 8C6 × ( 3 ! )( 2 ! )  ( 4 ! )( 2 ! )

BANSAL CLASSES Private Ltd. ‘Bansal Tower’, A-10, Road No.-1, I.P.I.A., Kota-05 Page # 12
PERMUTATION AND COMBINATION

(iii) |A|A|I|N|A|T|I|O|N|
Out of 10 gaps select 4
9!
Total ways = 10C4 ×
( 3 ! )( 2 ! )( 2 ! )

(iv) Total – all four S together


13 ! 10 !
 =?
( 4 ! )( 3 ! )( 2 ! )( 2 ! ) ( 3 ! )( 2 ! )( 2 ! )

 Consider S S S S as one string.

Illustration :
How many words can be formed using all the letters of the word HONOLULU if no two alike
letters are together.
Sol. Let A represent's ways when OO together, B when LL together, C when UU together.
Required ways = Total ways – [When all three alike letters together + when 2 alike letters together
+ when one alike letter together]
= Total ways – [n(E3) + n(E2) + n(E1)] …(i)
8!
Total ways = = 5040
( 2 ! )( 2 ! )( 2! )
(a) n(E3) = A  B  C (Region 7)
i.e., H N OO LL UU
n(E3) = 5! = 120

(b) n (E2) = 3 [(A  B) – (A  B  C)] or [Region 4 + 5 + 6]


n(E3)

6!  OO
= 3   5 !  = 720 1 B LL
 2!  A
4 2
i.e., H OO N LL UU 7
6
5
(c) n (E1) = 3 [ A – {(A  B) + (A  C)} + (A  B  C)]
 7!  6!  
    2   5!  1980 C 3
 ( 2! )( 2! )  2!   UU
Put in 1 st

Required ways = 5040 – [120 + 720 + 1980] = 2220 Ans.

BANSAL CLASSES Private Ltd. ‘Bansal Tower’, A-10, Road No.-1, I.P.I.A., Kota-05 Page # 13
PERMUTATION AND COMBINATION

Illustration :
Total number of ways of forming 6 letter word from the letters of word "PROPORTION".
Sol. There are
P  2: R  2, O  3,T  1, I  1, N1

Category Process of Selection Selection Arrangement


(1) 2 alike, 4 different (a) 2 alike PP, RR, OOO
6!
 3C 1 3C 5
1 × C4 = 15 15 × 2 !

(b) 4 different from 5 possible


option  5C4

(2) 3 alike, 3 different (a) 3 alike OOO  1C1


6!
(b) 3 different out of 5 1C 5
1 × C3 = 10 10× 3 !

different option  5C3

(3) 2 alike of one kind (a) 2 alike of one and 2 alike of


6!
2 alike other kind other PP, RR, OOO  3C2 3C 4
2 × C2 = 18 18× 2 ! 2 !

& 2 different (b) 2 out of 4 different  4C2

6!
(4) 3 alike of one kind (a) OOO, PP, RR  1C1 × 2C1 2C 4
1 × C1 = 8 8 × 3! 2 !

+ 2 a like of another (b) 2 out of 4 different  4C2


kind + 1 different
(5) 2 alike, 2 alike PP, RR, OOO
6!
+ 2 alike  1C 1 × 1C 1 × 1C 1 1C
1 = 1 1× 2 ! 2 ! 2 !

(6) All 6 different 6 possible option


P, R, O, T, I, N  6C6 6C
6 =1 6!

Total Number of Number of


selection = 53 words = 11130

Then total number of words formed = 11130

BANSAL CLASSES Private Ltd. ‘Bansal Tower’, A-10, Road No.-1, I.P.I.A., Kota-05 Page # 14
PERMUTATION AND COMBINATION

PERMUTATION OF THE OBJECTS TAKEN SOME AT A TIME :


Illustration :
How many 5 lettered words can be formed using the letters of the words "INDEPENDENCE".
Sol. There are E  4, N  3, D  2, P  1, I  1, C  1
Category Selection Arrangement
1  5 C1 5!
4 alike & diff.  5  25
eg : EEEED 4!
2
C1  5 C 2 5!
3 alike & 2 diff.  20  400
EEEDD 3!
2
3 alike & 2 alike of diff. kind C1  2 C 2  4 5!
 4  40
EEEDD 3!  2 !
3
C 2  3 C 3  30 5!
2 alike & 3 diff.  30  1800
EEND1 2!

2 alike  2 other alike 3


C 2  4  12 5!
 12  360
and 1 diff. EENND 2!  2!
6
all five diff. C 5  16 6  5!  720
EED1PC

CIRCULAR PERMUTATION:
When object are different
Permutation of objects in a row is called as linear permutation. If we arrange the objects along a closed
curve it is called as circular permutation.
Thus in, circular permutation, we consider one object fixed and the remaining objects are arranged as in
the case of a linear arrangements.
Theorem-I :
The number of circular permutation of n distinct objects is (n – 1) !
Proof :-Consider 5 objects A, B, C, D, E to be arranged around a closed curve is called circular permutation.
A E D C B
1 5 4 3 2
E 5 2 B D 4 1 A C 3 5 E B 2 4 D A 1 3 C

4 3 3 2 2 1 1 5 5 4
D C C B B A A E E D

All are Same


Let the total number of circular permutation be x.Above circular permutation is equivalent to 5 linear
permutations given byABCDEF, EABCD, DEABC, CDEAB, BCDEA
that is one circular permutation is equivalent to 5x linear permutation given by
5! 5.5  1!
x.5=5! ; x= = = (5 - 1) !
5 5

BANSAL CLASSES Private Ltd. ‘Bansal Tower’, A-10, Road No.-1, I.P.I.A., Kota-05 Page # 15
PERMUTATION AND COMBINATION

Similarly for n objects nx = n !


n!
x  (n  1)!
n
(i) These n things taken all at a time and arranged along circle in (n – 1) ! ways
(ii) Taken r things out of n things at a time and arranged along circle in nCr  (r – 1) ! ways.
Note :
In the above theorem anti–clockwise and clockwise order of arrangements are considered as distinct
permutations.
Theorm-II :
If anticlockwise and clockwise are considered to be same total number of circular permutation given by
n  1!
.
2
If we arrange flowers or garland beads in a neckless then there is no distinction between clockwise &
anticlockwise direction.
Turn over
Flip to right

Clock wise Counter clockwise Both Same


Note:-
(i) If we have n different things taken r at a time in form of a garland or neckless
C r  r  1!
n
Required number of arrangements = .
2
(ii) The distinction between clockwise and anticlockwise is ignored when a number of people have to be
seated around a table so as not to have the same neighbours.
Illustration :
Find the number of ways in which 9 people can be seated on a round table so that all shall not
have the same neighbours in any 2 arrangements.
Sol. For same neighbour, clockwise and anticlockwise arrangements are same.
So total number of ways will be arrangement of 9 people taken clockwise and anticlockwise same
( 9  1 )! 8!
and equal to 
2 2 A A
B I I B

C H H C

D G G D
E F F E

Same

BANSAL CLASSES Private Ltd. ‘Bansal Tower’, A-10, Road No.-1, I.P.I.A., Kota-05 Page # 16
PERMUTATION AND COMBINATION

Illustration :
There are n intermediate stations on a railway line from one terminus to another. In how many
ways can the train stop at 3 of these intermediate stations if
(a) all the three stations are consecutive
(b) at least two of the stations are consecutive
(c) no two of these stations are consecutive.
Sol.
(a) The number of triples of consecutive stations, viz.
S1S2S3, S2S3S4, S3S4S5, ……, Sn–2Sn–1Sn
is (n – 2).
(b) The total number of consecutive pair of stations, viz.
S1S2, S2S3 , …… Sn–1Sn
is (n – 1).
Each of the above pair can be associated with a third station in (n – 2) ways. Thus, choosing a
pair of stations and any third station can be done in (n – 1) (n – 2) ways. The above count also
includes the case of three consecutive stations. However, we can see that each such case has
counted twice. For example, the pair S4S5 combined with S6 and the pair S3S6 combined with
S4 are identical.
Hence, subtracting the excess counting, the number of ways which three stations can be chosen
so that at least two of them are consecutive
= (n – 1) (n – 2) – (n – 2) = (n – 2)2.
(c) Without restriction, the train can stop at any three stations in nC3 ways.
Hence, the number of ways the train can stop so that no two stations are consecutive
n( n  1 )( n  2 )
= nC3 – (n – 2)2 = – (n – 2)2
1·2·3
 n  n  6 n  12  ( n  2 )( n  3 )( n  4 )
2
= (n – 2)  =
 = n–2C3
 6  6
Illustration :
n different things are arranged in a circle. In how many ways can three objects be selected if
(a) all the three objects are consecutive
(b) at least two of the objects are consecutive
(c) no two objects are consecutive
Sol.
(a) The number of triaples of consecutive objects, viz. a1
an a2
a1a2a3, a2a3a4, …… ana1a2 an–1 a3
is n
(b) The total number of consecutive pair of objects, viz.
a1a2a3, a2a3, …… ana1 ar+1 ar–1
ar
is n
Each of the above pair can be associated with a third objects as
a1a2a3, a1a2a4, a1a2a5, …… a1a2an–1
in (n – 3) ways.
Note that a1 a 2a n has not been counted since it will be counted when we write an a 1a 2 .
Thus, choosing a pair of objects and any third object can be done in n(n – 2) ways.
Hence, the number of ways of selecting thee objects so that at least two of them are consecutive.
= n (n – 3)

BANSAL CLASSES Private Ltd. ‘Bansal Tower’, A-10, Road No.-1, I.P.I.A., Kota-05 Page # 17
PERMUTATION AND COMBINATION

(c) Without restriction, three objects can be selected in nC3 ways


Hence, the number of ways of selecting three objects so that no two of them are consecutive
= nC3 – n (n – 3)

Illustration :
How many hexagon's can be constructed by joining vertices of a guindecagon (15 side's polygon)
if none of the sides of Hexagon is also the sides of quindecagon.
Sol. Step (i) Select the initial vertex say '1' (in 15C1 ways)

(ii) Now 2 and 15 cannot be selected. From the 15 1


2
remaining vertices 3 to 14 (twelve) we have 14
to select 5 more for our hexagon. 13 3

(iii) Symbolise the vertices to be taken by 12 4


S S S S S and the vertices not be 11 5
taken 10 6
(7 in this case) by X X X X X X X. 9 8 7

(iv) Identify the gaps between these crosses (8 in this case) and select
4 6 7 9 10 12 14
X X X X X X X
S S S S S any five out of these gaps in 8C5 ways.
3 5 8 11 13

(v) Serial number are to be allotted either to S or to 'X' whichever comes earlier. In
the present problem the vertices corresponding to one selection are 3, 5, 8, 11, 13
and the hexagon as shown
(vi) For each selection we therefore have a hexagon with two non consecutive vertices.

(vii) Number of hexagons = 15C1 × 8C5. However this particular hexagon 1, 3, 5, 8,


11, 13 will occur 6 times when we select the initial vertex as 3 or 5 or 8 or 11 or 13.
Hence our answer is 6 times more.
15  8C5 15  56
(viii) Required number of hexagons =  = 140 Ans.
6 6
Alternative Solution :
As in linear let us open this chain to have 1, 2, 3, …… 13, 14, 15 O O O O O O to be selected ;
| X | X | X | X | X | X | X | X | X | not to be selected number of ways = 10C6
Required number of ways = 10C6 – number of ways when 1 and 15 are included, since in circular
these become consecutive
Now if 1 and 15 are already selected , 2 and 14 cannot be taken. Remaining vertices are
3, 4, 5, ……, 11, 12, 13 (11); O O O O (4); | X | X | X | X | X | X | (7)
Cases to be rejected = 8C4
Required number of ways = 10C6 – 8C4  10C4 – 8C4  210 – 70 = 140 Ans.

Case-II : Circular permutation which objects are alike.

BANSAL CLASSES Private Ltd. ‘Bansal Tower’, A-10, Road No.-1, I.P.I.A., Kota-05 Page # 18
PERMUTATION AND COMBINATION

Illustration :
In how many ways letter's word ALLAHABAD can be arranged in a circle. A
D L
Sol. There are four A's & two L's
A L
( 9  1 )! 8!
Required ways = = B A
4 ! 2! 4! 2!
A H

TOTAL NUMBER OF COMBINATIONS OR SELECTION OR COLLECTION :


We know that
(1 + x)n = nC0 + nC1x + nC2x2 + nC3x3 +........ + nCrxr + ..... + nCn yn.
Now replace x by 1
then 2n = nC0 + nC1 + nC2 + nC3 + …… + nCn

Case-I : Selection of one or more things out of n things. When all the things are different total number of
selections.
One thing can be selected in nC1 ways
Two things can be selected in nC2 ways
Three things can be selected in nC3 ways

n things can be selected in nCn ways
Total ways = nC1 + nC2 + nC3 + …… + nCn = 2n – 1

Illustration :
Sanjeev has 7 friend's. In how many ways can be invite one or more of then to dinner.
Sol. 7C1 + 7C2 + 7C3 …… 7C7 = 27 – 1

Case-II : The number of selecting zero or more things out of N identical things is N + 1
Proof :
Selecting none thing = 1 way
Selecting 1 thing = 1 ways
Selecting 2 things = 1 way

Selecting n things = 1 way
Total number of ways = 1 + 1 + 1 ……(n + 1) times = (n + 1) Ans.
Case-III : Number of ways selecting one or more things out of which p are alike of one kind, q are alike of
second kind, r alike of third kind, while s are different is (p + 1) (q + 1) (r + 1) 2S – 1
Proof :
Selecting none thing (out of p alike things) = 1 way
Selecting 1 thing (out of p alike things) = 1 ways
Selecting 2 things (out of p alike things) = 1 way

Selecting p things (out of p alike things) = 1 way
Total number of ways = 1 + 1 + 1 ……(p + 1) times = (p + 1) Ans.
Similarly for q alike, total ways = q + 1
Similarly for r alike, total ways = r + 1
For s different things total ways of selection will be 2S , i.e. any item is selected or not.
So total number of required ways = (p + 1) (q + 1) (r + 1) 2S – 1
(1 is subtracted when no item is selected)

BANSAL CLASSES Private Ltd. ‘Bansal Tower’, A-10, Road No.-1, I.P.I.A., Kota-05 Page # 19
PERMUTATION AND COMBINATION

Illustration :
Find the number of ways in which one or more letter be selected from the letters "AAAABBCCCDEF"
Sol. Total number of ways = (4 + 1) (2 + 1) (3 + 1) 23 – 1 = 479
A B C DEF

Illustration :
It is given that 4 Apples, 3 Mangoes, 2 Bananas, 2 Oranges, consider the following cases.
Case-I: Fruits of same species are alike and rests are different ,then
(i) Find the number of ways, atleast one fruit is selected.
(ii) Find the number of ways, atleast one fruit of each kind is selected.

Case-II: Fruits of same species are different and rests are also different , then
(i) Find the number of ways, atleast one fruit is selected.
(ii) Find the number of ways, atleast one fruit of each kind is selected.
Sol. Case-I :
(i) Apples can be selected in (4 + 1) ways
Total number of ways = (4 + 1) (3 + 1) (2 + 1) (2 + 1) – 1 = 179
(ii) Since we need atleast one fruit of each kind, Apples can be selected in 4 ways.
So total number of ways = 4 × 3 × 2 × 2 = 48
Case-II :
(i) Since fruits of same kind are different.
Apples can be selected in 24 ways
Total number of ways = 24 × 23 × 22 × 22 – 1 = 2047

(ii) Since we need atleast one fruit of each kind, Apples can be selected in 24 – 1 ways.
So total number of ways = (24 – 1)× (23 –1) × (22 –1) ×(22 – 1)

PROBLEMS BASED ON NUMBER THEORY :


Note that every natural number except 1 has atleast 2 divisors. If it has exactly two divisors then it is
called a prime. System of prime numbers begins with 2.All primes except 2 are odd.All primes are odd
but all odds are not prime.Anumber having more than 2 divisors is called a composite. 2 is the only even
number which is not composite.Apair of natural numbers are said to be relatively prime or coprime if
their HCF is one. For two natural numbers to be relatively it is not necessary that one or both should be
prime. It is possible that they both are composite but still coprime. eg. 4 and 25. Note that 1 is neither
prime nor composite however it is coprime with every other natural number.Apair of primes are said to
be twin if their non-negative difference is 2 e.g. 3 & 5 ; 5 & 7 e.t.c.

Total number of combinations in different cases :

(a) The number of combinations of n different things taking some or all (or atleast one) at a time
= nC1+ nC2 + .....+ nCn = 2n – 1
(b) The number of ways to select some or all out of (p + q + r) things where p are alike of first kind, q are
alike of second kind and r are alike of third kind is = (p +1) (q +1) ( r +1) – 1
(c) The number of ways to select some or all out of (p + q + t) things where p are alike of first kind, q are
alike of second kind and remaining t are different is = (p +1) (q +1) 2t – 1

BANSAL CLASSES Private Ltd. ‘Bansal Tower’, A-10, Road No.-1, I.P.I.A., Kota-05 Page # 20
PERMUTATION AND COMBINATION

Number of divisors and their sum :


  
(a) Every natural number N can always be put in the form N = p1 1 , p 2 2 ... p k k where p1, p2, ... pk are
distinct primes and 1, 2, ... , k are non-negative integers.
(b) If N = p11 , p 2 2 ... p k k then number of divisor of N is equivalent of number of ways of selecting zero
or more objects from the groups of identical objects, (p1, p1, ..., 1 times) (p2, p2, ... 2 times),
(pk, pk, ... , k times) = (1 + 1) (2 + 1) ... (k + 1) which includes 1 and N also.
(c) All the divisors excluding 1 and N are called proper divisors.
Also number of divisor of N can be seen as number of different terms in the expansion of
(p10 + p11 + p12 + ... + p11 ) × (p20 + p21 + p22 + ... + p 2 2 ) × ..... × (1 + pk + pk2 + ... + p k k )
Hence, sum of the divisors of N is
(1 + p1 + p12 + ... + p11 ) × (1 + p2 + p22 + ... + p 2 2 ) .... (1 + pk + pk2 + ... + p k k )
 1  1  1
p1 1  1 p 2 2  1 pk k  1
= ...
p1  1 p2  1 pk  1
1
(d) Number of ways of putting N as a product of two natural numbers is (a + 1) (a2 + 1).....(ak + 1) if
2 1
N is not a perfect square.
1
If N is a perfect square, then this is [(a + 1)(a2 + 1)......(ak + 1) + 1].
2 1
Illustration :
Consider the number N = 25 × 34 × 57 × 72 ,
Now answer the following question.
(i) Total number of divisor,
(ii) Number of proper divisor
(iii) Number of odd divisor
(iv) Number of even divisor.
(v) Number of divisors divisible by 5.
(vi) Number of divisors divisible by 10.
(vii) Number of divisors divisible by 2 but not by 4.
(viii) Sum of all divisors
(ix) Sum of even divisors
(x) Sum of odd divisors
(xi) Sum of divisor of the form (4n + 2), n  N (4n + 2)  Even number but not divisible
by 4, so exactly one 2.
(xii) Number of ways in which N can be resolved as product of two divisor.
Sol.
(i)
50
0
2 51
21 30 70
52
31
22 5
3

2 3 32 5 7 71
23 54
33
24 55 72
34
25 56
57

BANSAL CLASSES Private Ltd. ‘Bansal Tower’, A-10, Road No.-1, I.P.I.A., Kota-05 Page # 21
PERMUTATION AND COMBINATION

Number of divisor = (5 + 1) (4 + 1) (7 + 1) (2 + 1)
= 6 × 5 × 8 × 3 = 720
(ii) Proper divisor will be other than 1 and number itself.
So proper divisor = 720 – 2 = 718.
(iii) Number of odd divisor can be obtained by choosing 34 × 57 × 72
Which can be formed by (4 + 1) (7 + 1) × (2 + 1) = 120 ways
(iv) It can be obtain by selecting atleast one '2' in 25 · 34 · 57 · 72
which can be done in 5 × (4 + 1) × (7 + 1) × (2 + 1) = 600
(v) Atleast one 5 must be there in 25 × 34 × 57 × 72
(5 + 1) (4 + 1) (7) (2 + 1) = 630
(vi) For divisibility by 10 number most be divisible by 2 & 5.
So total divisor must contain atleast one 2 and atleast one 5 in 25 × 34 × 57 × 72
So that ways = 5 × (4 + 1) × (7) × (2 + 1) = 525
(vii) Exact one 2 should be there
Total ways = 1 × 5 × 8 × 3 = 120
(viii) It can be obtained from
(20 + 21 + 22 + 23 + 24 + 25) (30 + 31 + 32 + 33 + 34)
(50 + 51 + 52 + 53 + 54 + …… + 57) (70 + 71 + 72)
( 35  1 ) ( 58  1 ) ( 7 3  1 )
= (26 – 1) ×  
2 4 6
(ix) At least one 2 must be there
Sum = (21 + 22 + …… + 25) (30 + 31 + …… + 34) (50 + 51 + …… + 57) (70 + 71 + 72)
( 35  1 ) ( 5 8  1 ) ( 7 3  1 )
Sum = 2 · (25
– 1) ×  
2 4 6
(x) No 2 can be selected
Sum = (30 + 31 + 32 + 33 + 34) (50 + 51 + …… + 57) (70 + 71 + 72)

(xi) Sum = (2) (30 + 31 + 32 + 33 + 34) (50 + 51 + …… + 57) (70 + 71 + 72)


( 35  1 ) ( 58  1 ) ( 7 3  1 ) ( 35  1 )( 5 8  1 )( 7 3  1 )
=2×   =
2 4 6 24
(xii) If N = pa × qb × rc …… p, q are prime number & a, b are natural number.
(Total num ber of divisors)
I=
2
where I is the number of required ways if N is not a perfect square.
(Total number of divisors  1 )
I=
2
where I is the number of required ways if N is a perfect square.
For this problem, N = 25 × 34 × 57 × 72 is not a perfect square
(Total num ber of divisors) 720
So I= = = 360 Ans.
2 2

BANSAL CLASSES Private Ltd. ‘Bansal Tower’, A-10, Road No.-1, I.P.I.A., Kota-05 Page # 22
PERMUTATION AND COMBINATION

Illustration :
Find the number of permutation of 6 digits from the set {1, 2, 3, 4, 5, 6} where each digit is to be
used exactly once, so that the chosen permutation changes from increasing to decreasing or
decreasing to increasing at most once e.g. the strings like 1 2 3 4 5 6, 6 5 4 3 2 1, 1 2 6 5 4 3 and
6 3 2 1 4 5 are acceptable but strings like 1 3 2 4 5 6 or 6 5 3 2 4 1 are not.

Sol. {1, 2, 3, 4, 5, 6}
Case-1: up up up up up 6 5C
0
Case-2: up up up up 6 down 5C
1
Case-3: up up up 6 down down 5C
2
Case-4: up up 6 down down down 5C
3
Case-5: up 6 down down down down 5C
4
Case-6: 6 down down down down down 5C
5
Case-7: down down down down 1 up 5C
1
Case-8: down down down 1 up up 5C
2
Case-9: down down 1 up up up 5C
3
Case-10: down 1 up up up up 5C
4
Total = 62 Ans.
Aliter: 6C1 × 2 + 6C2 × 2 + 6C3 = 12 + 30 + 20 = 62 Ans.

SUMMATION OF NUMBERS (3 DIFFERENT WAYS) :


(a) Sum of all the numbers greater then 10000 formed by the digits 1,3,5,7,9 if no digit being repeated.

Method - 1 : All possible numbers = 5! = 120


If one occupies the units place then total numbers = 24.
Hence 1 enjoys units place 24 times
|||ly 1 enjoys each place 24 times
Sum due to 1 = 1 × 24 ( 1+ 10 + 102 + 103 + 104)
|||ly Sum due to the
digit 3 = 3 × 24 ( 1 + 10 + 102 + 103 + 104)
      
2 3 4
Required total sum = 24 ( 1 + 10 + 10 + 10 + 10 ) ( 1+ 3 + 5 + 7 + 9 )

Method –2 : In 1st column there are twenty four 1’s , Twenty four 3’s & so on and their sum is
= 24 × 25 = 600
Hence add. in vertical column normally we get = 6666600
5 th 2nd 1st
X X X X X
X X X X X
    
    
X X X X X
____________________________
666 6 6 0 0 = 6666600

BANSAL CLASSES Private Ltd. ‘Bansal Tower’, A-10, Road No.-1, I.P.I.A., Kota-05 Page # 23
PERMUTATION AND COMBINATION

Method–3 : Applicable only if the digits used are such that they have the same
common difference. (valid even if the digits are repeating)
Writing all the numbers in ascending order of magnitude
S = ( 13579 + 13597 + ........... + 97513 + 97531)
S = ( 13579 + 99531) + (13597 + 97513) + ....
= (111110) 60 time = 6666600 Ans
n
S= (l + L) where n = number of numbers, l = smallest, L = Largest
2
Illustration :
Find sum of all the numbers greater than 10000 formed by the digit 0, 1, 2, 4, 5, no digit being
repeated.
Sol. Using all the given digits we can form a five digit number except when zero is at first place.
So to find the sum of all the possible five digit number
= (Sum of all possible arrangement) – (Sum of all the arrangements when zero is at first place)
5!
 5 different digits can be arranged in 5! ways so each digit will appear at every place = times
5
i.e. 24 times
Sum of all digits at unit place = 24 (0 + 1 + 2 + 4 + 5) × 100
Sum of all digits at ten's place = 24 (0 + 1 + 2 + 4 + 5) × 101
…………………………………………………………
Sum of all digits at 10000th place = 24 (0 + 1 + 2 + 4 + 5) × 104
In this way sum of all possible arrangement = 24 (0 + 1 + 2 + 4 + 5) [1 + 10 + 102 + 103 + 104]
when zero is at first place 4 digit number will be formed.
Each number will appear 6 times at every place.
Sum of all 4 digit number at unit place = 6 (1 + 2 + 4 + 5) × 100
Sum of all 4 digit number at ten's place = 6 (1 + 2 + 4 + 5) × 101
…………………………………………………………
Hence sum of all four digit numbers = 6 (1 + 2 + 4 + 5) (1 + 10 + 102 + 103)
Required sum = 24 [0 + 1 + 2 + 3 + 4 + 5] [1 + 10 + 102 + 103 + 104]
– 6 (1 + 2 + 4 + 5) (1 + 10 + 102 + 103)

Illustration :
Find the number of permutations of the digits 1, 2, 3, 4 and 5 taken all at a time so that the sum
of the digits at the first two places is smaller than the sum of the digits at the last two places.
Sol. Total = 120; a1 a2 a3 a4 a5
Number of ways in which a1 + a2 > a4 + a5 = number of ways in which a1 + a2 < a4 + a5
( 120 )  ( a1  a2  a4  a5 )*
 required number =
2
(* denotes the number of ways when a1 + a2 = a4 + a5)
Now a1 + a2 = a4 + a5 (1, 2, 3, 4, 5)
If a3 = 1, 43125 8 ways
a3 = 3, 15324 8 ways
a3 = 5, 41523 8 ways
If a3 = 2 / 4 / 6 not possible (think !)
( 120 )  24
 required number = = 48 Ans.
2
BANSAL CLASSES Private Ltd. ‘Bansal Tower’, A-10, Road No.-1, I.P.I.A., Kota-05 Page # 24
PERMUTATION AND COMBINATION

DISTRIBUTION OF ALIKE OBJECTS :

TYPE-1:
Total number of ways in which n identical coins can be distributed between p persons so that each
n  p  1!
person may get any number of coin is n+p–1Cp–1 = p  1! n !

Proof:
Let 6 identical coins can be distributed among 3 persons R|S|G

(False coins)
Out of eight places if we select any two 3
R S G
partitions are formed by 8C2 ways
Partition wall
Illustration :
Find number of ways in which 30 mangos can be distributed among 5 persons.
Sol. 30+5–1C5–1 = 34C4

TYPE-2:
Total number of ways in which n identical items can be distributed among p persons such that each of
them receive at least one item n–1Cp–1.
Illustration :
Find total number of ways of distributing 7 identical computers to R|S|G. So that each receive atleast one
computer
Sol. 7–1C3–1 = 6C2.
Illustration :
Find number of natural solutions of equation x + y + z = 102, where x, y, z  N.
Sol. x + y + z = 102consider x, y, z as 3 beggars and 102 as coins, give 1 coin to each equation becomes
X + Y + Z = 99. X= x + 1, Y = y + 1, Z = z + 1.
Now distributed 99 coins without constraints total ways 99+2C2 = 101C2.
Imp. Point :
Number of different terms in a complete homogeneous expression of degree m in n variables
is equivalent to distribution of m identical coins among n beggars.
If expression is (x1 + x2 + x3 ....... + xn)m number of terms m+n–1Cn–1.
Illustration :
Find number of different terms in expansion of (x+y+z)10 .
Sol. 10+3–1C3–1 = 12C2.

BANSAL CLASSES Private Ltd. ‘Bansal Tower’, A-10, Road No.-1, I.P.I.A., Kota-05 Page # 25
PERMUTATION AND COMBINATION

Illustration :
Number of non-negative integral solution of the inequality x+y+z+t  30.
Sol. (x+y+z+t) + w = 30

(False beggar)
Required condition is equivalent to giving 30 coin’s to 5 beggar’s = 30+5–1C5–1 = 34C4.

Illustration :
Number of non-negative integral solutions of x1+ x2+ x3 + 4x4 = 20 will be
(A) 496 (B) 516 (C) 536 (D) 546
Sol. Here, clearly 0  x4  5, x1, x2, x3,  0 and x1 + x2 + x3 = 20 – 4x4
 r = 3 and n = 20 – 4x1
If x4 = 0 number of ways 20+3 – 1C3 – 1 = 22C2
If x4 = 1 number of ways 16+3 – 1C3 – 1 = 18C2 Similarly, if x4 = 2, 3, 4, 5, number of ways = 14C2,
10C , 6C , 2C respectively
2 2 2
 Total number of ways
= 22C2 + 18C2 + 14C2 + 10C2 + 6C2 + 2C2 = 536

MAXIMISE nCr :
n
r if n is even
 2
nC is maximum at 
r
 n 1 n 1
r  or if n is odd
2 2
e.g. 15C 12C is maximum when r = 6
r is maximum when r = 7 or 8, r

GRID PROBLEM :
Complete cartesian plane is partitioned by drawing line | | to x and y-axis equidistant apart like the lines
on a chess board. Then the
Illustration :
Number of ways in which an ant are can reach from (1, 1) to (4, 5) via shortest path.
Sol. Whatever may be the mode of travel of the ant; it has to traverse 3H (Horizontal) and 4V (Vertical)
paths.
7! 7
Hence required number of ways = = C3
4! 3!
(4,5)

(1,1)
O

Note : If there are n rational and m horizontal lines then there will be (n – 1) horizontal and (m – 1)
vertical paths.
BANSAL CLASSES Private Ltd. ‘Bansal Tower’, A-10, Road No.-1, I.P.I.A., Kota-05 Page # 26
PERMUTATION AND COMBINATION

DERANGEMENT :
If n things are arranged in a row, the number of a ways they can be deranged so that r things occupy
wrong places while (n – r) things occupy their original places, is
= nCn–r Dr
 1 1 1 n 1 
where Dr = r! 1       (1) 
 1! 2! 3! n ! 
If n things are arranged in a row, the number of ways they can be deranged so that none of them occupies
its original place, is
 1 1 1 n 1 
= nC0 Dn = n ! 1       (1) 
 1! 2! 3! n ! 
Aliter :
Dn = ways in which n things are arranged so that all n things occupy wrong places
= arranged without restriction
– ways in which 1 thing is in correct position while (n – 1) things are deranged
– ways in which 2 things are in correct position while (n – 2) things are deranged
…………………………
…………………………
– ways in which all n things are in correct position and there is no derangement
= n! – nC1 Dn–1 – nC2Dn–2 – …………… – nCnD0
n
= n! –  n Cr Dn r
r 1
Thus, we have
D0 = 0! = 1
D1 = 1! – 1C1D0 = 0
D2 = 2! – 2C1D1 – 2C2D0 = 1
D3 = 3! – 3C1D2 – 3C2D1 – 3C3D0 = 2
D4 = 4! – 4C1D3 – 4C2D2 – 4C3D1 – 4C4D0 = 9
D5 = 5! – 5C1D4 – 5C2D3 – 5C3D2 – 5C4D1 – 5C5D0 = 44
and so on.
Illustration:
A person writes letters to five friends and addresses the corresponding envelopes. In how many
ways can the letters be placed in the envelops so that
(a) all letters are in the wrong envelopes.
(b) at least three of them are in the wrong envelopes.
 1 1 1 1 1  5! 5! 5! 5!
Sol. (a) Required number of ways  5!  1          
 1! 2! 3! 4! 5!  2! 3! 4! 5!
(b) Required number of ways
n
 n! n C r D n r where n = 5
n 1
= 5C2D3 + 5C D
1 4 + 5C0D5
 1 1 1  1 1 1 1  1 1 1 1 1
= 10 × 3!  1     + 5 × 4!  1      + 1 × 5!  1      
 1! 2! 3!   1! 2! 3! 4!   1! 2! 3! 4! 5! 
= 10 (3 – 1) + 5 (12 – 4 + 1) + (60 – 20 + 5 – 1) = 20 + 45 + 44 = 109

BANSAL CLASSES Private Ltd. ‘Bansal Tower’, A-10, Road No.-1, I.P.I.A., Kota-05 Page # 27
PERMUTATION AND COMBINATION

SOME IMPORTANT RESULTS ABOUT POINTS :


If there are n points in a plane of which m (< n) are collinear, then

(a) Total number of different straight lines obtained by joining these n points is
nC – mC + 1
2 2

(b) Total number of different triangles formed by joining these n points is


nC – mC
3 3

(c) Number of diagonals in polygon of n sides is

nC
n ( n  3)
2–n i.e.
2
(d) If m parallel lines in a plane are intersected by a family of other n parallel lines. Then total number of
parallelograms so formed is

mC
mn ( m  1) ( n  1)
2 × nC 2 i.e.
4
(e) Number of triangles formed by joining vertices of convex polygon of n sides is nC3 of which
(i) Number of triangles having exactly two sides common to the polygon = n
(ii) Number of triangles having exactly one side common to the polygon = n(n – 4)
n ( n  4)( n  5)
(iii) Number of triangles having no side common to the polygon =
6

BANSAL CLASSES Private Ltd. ‘Bansal Tower’, A-10, Road No.-1, I.P.I.A., Kota-05 Page # 28
PERMUTATION AND COMBINATION

EXERCISE-1 (SPECIAL DPP)

SPECIAL DPP-1

ELEMENTARY PROBLEMS ON PERMUTATION & COMBINATION

NOTE: USE FUNDAMENTAL PRINCIPLE OF COUNTING & ENJOY DOING THE FOLLOWING.
Q.1 There are 6 roads between A & B and 4 roads between B & C.
(i) In how many ways can one drive from A to C by way of B?
(ii) In how many ways can one drive from A to C and back to A, passing through B on both trips ?
(iii) In how manyways can one drive the circular trip described in (ii) without using the same road more than
once.

Q.2 If the letters of the word “VARUN” are written in all possible ways and then are arranged as in a
dictionary, then the rank of the word VARUN is :
(A) 98 (B) 99 (C) 100 (D) 101

Q.3 How many natural numbers are their from 1 to 1000 which have none of their digits repeated.

Q.4 Afamily is being arranged in a line for a group photograph. If the family consists of a mother, a father,
a baby and five children, the number of arrangements that begin and end with a parent is
(A) 720 (B) 1440 (C) 5040 (D) 40320

Q.5 (i) Find the number of four letter word that can be formed from the letters of the word HISTORY.
(each letter to be used at most once)
(ii) How many of them contain only consonants?
(iii) How many of them begin & end in a consonant?
(iv) How manyof them begin with a vowel?
(v) How many contain the letters Y?
(vi) How many begin with T & end in a vowel?
(vii) How many begin with T & also contain S?
(viii) How many contain both vowels?

Q.6 If repetitions are not permitted


(i) How many 3 digit numbers can be formed from the six digits 2, 3, 5, 6, 7 & 9 ?
(ii) How many of these are less than 400 ?
(iii) How many are even ?
(iv) How many are odd ?
(v) How many are multiples of 5?

BANSAL CLASSES Private Ltd. ‘Bansal Tower’, A-10, Road No.-1, I.P.I.A., Kota-05 Page # 29
PERMUTATION AND COMBINATION

Q.7 Every telephone number consists of 7 digits. How many telephone numbers are there which do not
include any other digits but 2 , 3 , 5 & 7?

Q.8 How many of the arrangements of the letter of the word “LOGARITHM” begin with a vowel and end
with a consonant?

Q.9 Anew flag is to be designed with six vertical strips using some or all of the colours yellow, green, blue
and red. Then, the number of ways this can be done such that no two adjacent strips have the same
colour is
(A) 12 × 81 (B) 16 × 192 (C) 20 × 125 (D) 24 × 216

Q.10 How many four digit numbers are there which are divisible by 2.

Q.11 In a telephone system four different letter P, R, S, T and the four digits 3, 5, 7, 8 are used. Find the
maximum number of “telephone numbers” the system can have if each consists of a letter followed by a
four-digit number in which the digit may be repeated.

Q.12 Find the number of 5 lettered palindromes which can be formed using the letters from the English
alphabets.

Q.13 Number of ways in which 7 different colours in a rainbow can be arranged if green is always in the
middle.

Q.14 Find the number of ways in which 4 boys and 3 girls can be seated in a line if the terminal seats are
occupied by the boys.

Q.15 Numbers of words which can be formed using all the letters of the word "AKSHI", if each word begins
with vowel or terminates in vowel.

Q.16 A letter lock consists of three rings each marked with 10 different letters. Find the number of ways in
which it is possible to make an unsuccessful attempts to open the lock.

Q.17 How many 10 digit numbers can be made with odd digits so that no two consecutive digits are same.

Q.18 It is required to seat 5 men and 4 women in a row so that the women occupy the even places. How many
such arrangements are possible?

Q.19 Find the number of three digits or four digit even numbers that can be formed from the number
2, 3, 5, 6, 7 (without repetition).

Q.20 How many natural numbers are there with the property that they can be expressed as the sum of the
cubes of two natural numbers in two different ways.

BANSAL CLASSES Private Ltd. ‘Bansal Tower’, A-10, Road No.-1, I.P.I.A., Kota-05 Page # 30
PERMUTATION AND COMBINATION

SPECIAL DPP-2

Q.1 A student solves the equation nC2 = 10 using the following steps, but finds the solution yields decimal
answer and therefore he must not be correct which step did he make the mistake?
n!
(A) Step-1: = 10 (B) Step-2: n! = 10 (n – 2) !
(n  2)!
(C) Step-3: n (n – 1) (n – 2)! = 10 (n – 2)! (D) Step-4: n (n – 1) = 10  n2 – n – 10 = 0

Q.2 Number of natural numbers between 100 and 1000 such that at least one of their digits is 7, is
(A) 225 (B) 243 (C) 252 (D) none

Q.3 For some natural N , the number of positive integral ' x ' satisfying the equation ,
1 ! + 2 ! + 3 ! + ...... + (x !) = (N)2 is
(A) none (B) one (C) two (D) infinite

Q.4 All possible three digits even numbers which can be formed with the condition that if 5 is one of the digit,
then 7 is the next digit is :
(A) 5 (B) 325 (C) 345 (D) 365

Q.5 How many of the 900 three digit numbers have at least one even digit?
(A) 775 (B) 875 (C) 450 (D) 750
Q.6 The number of different seven digit numbers that can be written using only three digits
1, 2 & 3 under the condition that the digit 2 occurs exactly twice in each number is :
(A) 672 (B) 640 (C) 512 (D) none

Q.7 Out of seven consonants and four vowels, the number of words of six letters, formed by taking four
consonants and two vowels is (Assume that each ordered group of letter is a word):
(A) 210 (B) 462 (C) 151200 (D) 332640

Q.8 Find the number of natural numbers less than 1000 and divisible by 5 can be formed with the ten digits,
each digit not occuring more than once in each number.

Q.9 The set of values of r simultaneously satisfying the system of equations


P(5, r) = 2 · P(6, r – 1) and 5 · P(4, r) = 6 · P(5, r – 1), is
(A) an empty set (B) a singleton set
(C) a set consisting of two elements (D) a set consisting of three elements.

Q.10 Let P n denotes the number of permutations of n distinct things taken all at a time and

n 5  143   Pn 5 
xn = C4    (where n  N). The possible value of n for which xn is negative, can be
 96   Pn 3 
(A) 1 (B) 2 (C) 3 (D) 4

Q.11 10 people are sitting around a circular table, each one shaking a hand with everyone else except from the
people sitting on either side of him. Find the number of handshakers.

BANSAL CLASSES Private Ltd. ‘Bansal Tower’, A-10, Road No.-1, I.P.I.A., Kota-05 Page # 31
PERMUTATION AND COMBINATION

SPECIAL DPP-3

Q.1 Find the number of ways in which letters of the word VALEDICTORY be arranged so that the vowels
may never be separated.
Q.2 How manynumbers between 400 and 1000 (both exclusive) can be made with the digits 2,3,4,5,6,0 if
(a) repetition of digits not allowed. (b) repetition of digits is allowed.

Q.3 The interior angles of a regular polygon measure 150º each. The number of diagonals of the polygon is
(A) 35 (B) 44 (C) 54 (D) 78

Q.4 The number of ways in which 5 different books can be distributed among 10 people if each person can
get at most one book is :
(A) 252 (B) 105 (C) 510 (D) 10C5.5!

Q.5 The 9 horizontal and 9 vertical lines on an 8 × 8 chessboard form 'r' rectangles and 's' squares. The ratio
s
in its lowest terms is
r
(A) 1/6 (B)17/108 (C) 4/27 (D) none
Q.6 There are 720 permutations of the digits 1, 2, 3, 4, 5, 6. Suppose these permutations are arranged from
smallest to largest numerical values, beginning from 1 2 3 4 5 6 and ending with 6 5 4 3 2 1.
(a) What number falls on the 124th position?
(b) What is the position of the number 321546?

Q.7 Number of 4 digit numbers of the form N = abcd which satisfy following three conditions
(i) 4000  N < 6000 (ii) N is a multiple of 5 (iii) 3b<c6
is equal to
(A) 12 (B) 18 (C) 24 (D) 48

Q.8 Find the number of 3 digit numbers in which the digit at hundreath's place is greater than the other two
digit.

Q.9 Find the number of 6 digit numbers of the form abcdef if the digits satisfy the condition
a + b + c + d + e + f = a2 + b2 + c2 + d2 + e2 + f2.
Q.10 Number of permutations of 1, 2, 3, 4, 5, 6, 7, 8 and 9 taken all at a time, such that the digit
1 appearing somewhere to the left of 2
3 appearing to the left of 4 and
5 somewhere to the left of 6, is
(e.g. 815723946 would be one such permutation)
(A) 9 · 7! (B) 8! (C) 5! · 4! (D) 8! · 4!

Q.11 A convex polygon has 44 diagonals. The polygon is


(A) nonagon (B) decagon (C) undecagon (D) Dodecagon

Q.12 The number of ways in which 10 candidates A1,A2, .......,A10 can be ranked, so that A1 is always above
A2 is
10!
(A) (B) 8! × 10C2 (C) 10P
2 (D) 10C2
2
BANSAL CLASSES Private Ltd. ‘Bansal Tower’, A-10, Road No.-1, I.P.I.A., Kota-05 Page # 32
PERMUTATION AND COMBINATION

SPECIAL DPP-4

Q.1 Find the number of ways in which two squares can be selected from an 8 by 8 chess board
of size 1 × 1 so that they are not in the same row or in the same column.

Q.2 Number of different natural numbers which are smaller than two hundred million & using only the digits
1 or 2 is :
(A) (3) . 28  2 (B) (3) . 28  1 (C) 2 (29  1) (D) none

Q.3 5 Indian & 5 American couples meet at a party & shake hands . If no wife shakes hands with her own
husband & no Indian wife shakes hands with a male, then the number of hand shakes that takes place in
the party is
(A) 95 (B) 110 (C) 135 (D) 150

Q.4 The number of n digit numbers which consists of the digits 1 & 2 only if each digit is to be used atleast
once, is equal to 510 then n is equal to:
(A) 7 (B) 8 (C) 9 (D) 10

Q.5 Number of six digit numbers which have 3 digits even & 3 digits odd, if each digit is to be used atmost
once is ______ .

Q.6 Find the number of odd integers between 1000 and 8000 which have none of their digits repeated.

Q.7 Number of four digit numbers with all digits different and containing the digit 7 is
(A) 2016 (B) 1828 (C) 1848 (D) 1884

Q.8 A 5 digit number divisible by3 is to be formed using the numerals 0, 1, 2, 3, 4 & 5 without repetition. The
total number of ways this can be done is :
(A) 3125 (B) 600 (C) 240 (D) 216

Q.9 A committee of 5 is to be chosen from a group of 9 people. Number of ways in which it can be formed
if two particular persons either serve together or not at all and two other particular persons refuse to
serve with each other, is
(A) 41 (B) 36 (C) 47 (D) 76

Q.10 Aquestion paper on mathematics consists of twelve questions divided into three parts A, B and C, each
containing four questions . In how many ways can an examinee answer five questions, selecting atleast
one from each part .
(A) 624 (B) 208 (C) 2304 (D) none

Q.11 Consider 8 vertices of a regular octagon and its centre. If T denotes the number of triangles and S
denotes the number of straight lines that can be formed with these 9 points then T – S has the value equal
to
(A) 52 (B) 56 (C) 48 (D) 44

BANSAL CLASSES Private Ltd. ‘Bansal Tower’, A-10, Road No.-1, I.P.I.A., Kota-05 Page # 33
PERMUTATION AND COMBINATION

SPECIAL DPP-5

Q.1 The kindergarten teacher has 25 kids in her class . She takes 5 of them at a time, to zoological garden as
often as she can, without taking the same 5 kids more than once. Find the number of visits, the teacher
makes to the garden and also the number of of visits every kid makes.

Q.2 Number of ways in which 8 people can be arranged in a line ifA and B must be next to each other and
C must be somewhere behind D, is equal to
(A) 10080 (B) 5040 (C) 5050 (D) 10100

Q.3 In a certain algebraical exercise book there are 4 examples on arithmetical progressions, 5 examples on
permutation  combination and 6 examples on binomial theorem . Number of ways a teacher can select
for his pupils atleast one but not more than 2 examples from each of these sets, is ______.

Q.4 In how manydifferent ways a grandfather along with two of his grandsons and four grand daughters can
be seated in a line for a photograph so that he is always in the middle and the two grandsons are never
adjacent to each other.

Q.5 The number of 5 digit numbers such that the sum of their digits is even is
(A) 50000 (B) 45000 (C) 60000 (D) none

Q.6 A forecast is to be made of the results of five cricket matches, each of which can be win, a draw or a loss
for Indian team. Find
(i) the number of different possible forecasts
(ii) the number of forecasts containing 0, 1, 2, 3, 4 and 5 errors respectively

Q.7 The number of ways in which 8 distinguishable apples can be distributed among 3 boys such that every
boy should get atleast 1 apple & atmost 4 apples is K · 7P3 where K has the value equal to
(A) 14 (B) 66 (C) 44 (D) 22

Q.8 Awomen has 11 close friends. Find the number of ways in which she can invite 5 of them to dinner, if two
particular of them are not on speaking terms & will not attend together.

Q.9 A rack has 5 different pairs of shoes. The number of ways in which 4 shoes can be chosen from it so that
there will be no complete pair is
(A) 1920 (B) 200 (C) 110 (D) 80

Q.10 Find the number of different ways in which 8 different books can be distributed among 3 students, if each
student receives atleast 2 books.

BANSAL CLASSES Private Ltd. ‘Bansal Tower’, A-10, Road No.-1, I.P.I.A., Kota-05 Page # 34
PERMUTATION AND COMBINATION

SPECIAL DPP-6

Q.1 Find the number of permutations of the word "AUROBIND" in which vowels appear in an alphabetical
order.

Q.2 There are 10 different books in a shelf. If m denotes the number of ways of selecting 3 books when no
two of them are consecutive and n denotes the corresponding figure when exactly two are consecutive
then
(A) 8m = 7n (B) 6m = 5n (C) 7m = 5n (D) m = n

Q.3 If as many more words as possible be formed out of the letters of the word "DOGMATIC" then compute
the number of words in which the relative order of vowels and consonants remain unchanged.

Q.4 Number of ways in which 9 different toys be distributed among 4 children belonging to different age
groups in such a way that distribution among the 3 elder children is even and the youngest one is to
receive one toy more, is :
(5!) 2 9! 9!
(A) (B) (C) (D) none
8 2 3!(2!)3
Q.5 There are five different peaches and three different apples. Number of ways they can be divided into two
packs of four fruits if each pack must contain atleast one apple, is
(A) 35 (B) 65 (C) 60 (D) 30

Q.6 Let Pn denotes the number of ways in which three people can be selected out of ' n ' people sitting in
a row, if no two of them are consecutive. If , Pn + 1  Pn = 15 then the value of ' n ' is :
(A) 7 (B) 8 (C) 9 (D) 10

Q.7 Number of ways in which 7 green bottles and 8 blue bottles can be arranged in a row if exactly 1 pair of
green bottles is side by side, is (Assume all bottles to be alike except for the colour).
(A) 84 (B) 360 (C) 504 (D) none

Q.8 Ahas 3 maps and B has 9 maps.All the 12 maps being distinct. Determine the number of ways in which
they can exchange their maps if each keeps his initial number of maps.

Q.9 Number of three digit number with atleast one 3 and at least one 2 is
(A) 58 (B) 56 (C) 54 (D) 52

Q.10 Number of ways in which 3 mangoes, 3 apples and 2 oranges can be distributed in 8 children so that
every child gets exactly one fruit, is (Assume fruits of the same species to be alike.)
8!3! 8!2! 3 · 8! 8!
(A) 3! 3! 2!2! (B) 3! 3! 2!2! (C) 3! 3! 2!2! (D) 3! 3! 2!2!

Q.11 Shoma has ten apples, and wants to give them to three of her friends. Number of different ways can she
can given them is (Assume that the apples are all identical.)
(A) 36 (B) 63 (C) 66 (D) 1000

Q.12 Number of divisors of the number N = 23 · 35 · 57 · 79 which are perfect square is


(A) 24 (B) 60 (C) 119 (D) 120
BANSAL CLASSES Private Ltd. ‘Bansal Tower’, A-10, Road No.-1, I.P.I.A., Kota-05 Page # 35
PERMUTATION AND COMBINATION

Paragraph for question nos. 13 to 15


Consider the word W = MISSISSIPPI
Q.13 If N denotes the number of different selections of 5 letters from the word W = MISSISSIPPI then N
belongs to the set
(A) {15, 16, 17, 18, 19} (B) {20, 21, 22, 23, 24}
(C) {25, 26, 27, 28, 29} (D) {30, 31, 32, 33, 34}

Q.14 Number of ways in which the letters of the word W can be arranged if atleast one vowel is separated
from rest of the vowels
8!·161 8!·161 8!·161 8! 165
(A) 4!· 4!· 2! (B) 4 · 4!· 2! (C) 4!· 2! (D) 4!· 2! · 4!

 10! 
Q.15 If the number of arrangements of the letters of the word W if all the S's and P's are separated is (K)  
 4!· 4! 
then K equals
6 4 3
(A) (B) 1 (C) (D)
5 3 2
Q.16 There are nine different books on a shelf, four red and five are green. Number of ways in which it is
possible to arrange these books if
Column-I Column-II
(A) the red books must be together and green books together (P) (4) 6!
(B) the red books must be together whereas the green books may (Q) (8) 6!
or may not be together
(C) no two books of the same colour are adjacent (R) (20) 6!
(D) if the books are arranged on a round table and no two of the
three specified books are together (S) (24) 6!

Q.17 Find the number of five digit numbers that can be formed by using two 2's, three 3's, one zero and one 5.

SPECIAL DPP-7

Q.1 If all the words formed with letters of the word "POTATO" are arranged in alphabetical order as they
are done in a dictionary then the rank of the word "POTATO" is
(A) 44 (B) 103 (C) 104 (D) none

Q.2 A shelf contains 20 different books of which 4 are in single volume and the others form sets of 8, 5 and
3 volumes respectively. Number of ways in which the books may be arranged on the shelf, if the
volumes of each set are together and in their due order is
20!
(A) (B) 7! (C) 8! (D) 7 . 8!
8! 5! 3!

Q.3 If all the letters of the word "QUEUE" are arranged in all possible manner as they are in a
dictionary , then the rank of the word QUEUE is
(A) 15th (B) 16th (C) 17th (D) 18th
BANSAL CLASSES Private Ltd. ‘Bansal Tower’, A-10, Road No.-1, I.P.I.A., Kota-05 Page # 36
PERMUTATION AND COMBINATION

Q.4 Number of six digit numbers in which sum of the squares of the digits is 9 is
(A) 60 (B) 66 (C) 72 (D) 37

Q.5 All the five digit numbers in which each successive digit exceeds its predecessor are arranged in the
increasing order of their magnitude. The 97th number in the list does not contain the digit
(A) 4 (B) 5 (C) 7 (D) 8

Q.6 Find the number of combination of 16 things, 8 of which are alike and the rest different, taken 8 at a time.

Q.7 The number of different ways in which five 'dashes' and eight 'dots' can be arranged, using only seven of
these 13 'dashes' & 'dots' is :
(A) 1287 (B) 119 (C) 120 (D) 1235520

Q.8 An old man while dialing a 7 digit telephone number remembers that the first four digits consists of one
1's, one 2's and two 3's. He also remembers that the fifth digit is either a 4 or 5 while has no memorising
of the sixth digit, he remembers that the seventh digit is 9 minus the sixth digit. Maximum number of
distinct trials he has to try to make sure that he dials the correct telephone number, is
(A) 360 (B) 240 (C) 216 (D) none

Q.9 There are 2n white and 2n red counters. Counters are all alike except for the colour. If the number of
ways in which they can be arranged in a line so that they are symmetric with respect to a central mark
is 70 then n equals to
(A) 3 (B) 4 (C) 5 (D) 6

Paragraph for Question Nos. 10 to 12


16 players P1, P2, P3,.......P16 take part in a tennis tournament. Lower suffix player is better than any
higher suffix player. These players are to be divided into 4 groups each comprising of 4 players and the
best from each group is selected for semifinals.
8
Q.10 Number of ways in which 16 players can be divided into four equal groups, is K  (2r  1) ,
r 1
where K equals
35 35 35 35
(A) (B) (C) ` (D)
27 24 52 6

Q.11 Number of ways in which they can be divided into 4 equal groups if the players P1, P2, P3 and P4 are in
different groups, is :
(11)! (11)! (11)! (11)!
(A) (B) (C) (D)
36 72 108 216

Q.12 Number of ways in which these 16 players can be divided into four equal groups, such that when the
12!
best player is selected from each group, P6 is one among them, is (k) . The value of k is :
(4!)3
(A) 36 (B) 24 (C) 18 (D) 20

BANSAL CLASSES Private Ltd. ‘Bansal Tower’, A-10, Road No.-1, I.P.I.A., Kota-05 Page # 37
PERMUTATION AND COMBINATION

Paragraph for question nos. 13 to 15


Different words are formed by arranging the letters of the word "SUCCESS".
Q.13 The numbers of words in which C are together but S's are separated, is
(A) 120 (B) 96 (C) 24 (D) 420

Q.14 The number of words in which no two C's and no two S are together is
(A) 120 (B) 96 (C) 24 (D) 180

Q.15 The number of words in which the consonants appear in alphabatic order is
(A) 42 (B) 40 (C) 420 (D) 280

Q.16 How many different arrangements are possible with the factor of the term a2b4c5 written at full length.

Q.17 Find the number of 4 digit numbers starting with 1 and having exactly two identical digits.

Q.18 Number of ways in which 5 A's and 6 B's can be arranged in a row which reads the same backwards
and forwards, is

Q.19 In a plane a set of 8 parallel lines intersects a set of n parallel lines, that goes in another direction, forming
a total of 1260 parallelograms . Find the value of n.

SPECIAL DPP-8

Q.1 Number of different words that can be formed using all the letters of the word "DEEPMALA" if two
vowels are together and the other two are also together but separated from the first two is :
(A) 960 (B) 1200 (C) 2160 (D) 1440

Q.2 In a unique hockey series between India & Pakistan, they decide to play on till a team wins 5 matches .
The number of ways in which the series can be won by India, if no match ends in a draw is :
(A) 126 (B) 252 (C) 225 (D) none

Q.3 Number of cyphers at the end of 2002C is


1001
(A) 0 (B) 1 (C) 2 (D) 200

Q.4 Three vertices of a convex n sided polygon are selected. If the number of triangles that can be constructed
such that none of the sides of the triangle is also the side of the polygon is 30, then the polygon is a
(A) Heptagon (B) Octagon (C) Nonagon (D) Decagon

Q.5 There are 12 guests at a dinner party . Supposing that the master and mistress of the house have fixed
seats opposite one another, and that there are two specified guests who must always, be placed next to
one another ; the number of ways in which the company can be placed, is:
(A) 20 . 10 ! (B) 22 . 10 ! (C) 44 . 10 ! (D) none

Q.6 Let Pn denotes the number of ways of selecting 3 people out of ' n ' sitting in a row, if no two of them
are consecutive and Qn is the corresponding figure when they are in a circle . If Pn  Qn = 6 , then
' n ' is equal to :
(A) 8 (B) 9 (C) 10 (D) 12

BANSAL CLASSES Private Ltd. ‘Bansal Tower’, A-10, Road No.-1, I.P.I.A., Kota-05 Page # 38
PERMUTATION AND COMBINATION

Q.7 There are counters available in 3 different colours. The counters are all alike except for the colour.
Find the total number of arrangements consisting of 5 counters, assuming sufficient number of counters
of each colour, if each arrangement consists of counters of every colour.
(A) 90 (B) 150 (C) 540 (D) 196

The question given below contains STATEMENT-1 (Assertion) and STATEMENT-2 (Reason).
Question has 4 choices (A), (B), (C) and (D), out of which ONLY ONE is correct. Choose the
correct alternative.

Q.8 Statement 1: The sum 40C0 · 60C10 + 40C1 · 60C9 + ......... + 40C10 · 60C0 equals 100C10.
Statement 2: Number of ways of selecting 10 students out of 40 boys and 60 girls is 100C10.
(A) Statement-1 is true, statement-2 is true and statement-2 is correct explanation for statement-1.
(B) Statement-1 is true, statement-2 is true and statement-2 is NOT the correct explanation for statement-1.
(C) Statement-1 is true, statement-2 is false.
(D) Statement-1 is false, statement-2 is true.

Q.9 Define a 'good word' as a sequence of letters that consists only of the letters A, B and C and in which A
never immidiately followed by B, B is never immediately followed by C, and C is never immediately
followed byA. If the number of n-letter good words are 384, find the value of n.

Q.10 Six married couple are sitting in a room. Find the number of ways in which 4 people can be selected so that
(a) they do not form a couple (b) they form exactly one couple
(c) they form at least one couple (d) they form atmost one couple

Q.11 Column-I Column-II


(A) Number of four elements subsets of the set {1, 2, 3, 4, 5, 6, 7} (P) 6
which has 1 as an element but do not have 7 as an element, is
(B) Out of 12 points, p point are collinear (0 < p < 12). (Q) 7
If the number of triangles formed with vertices at these points is 185,
then the value of p, is equal to (R) 10
(C) Number of ways in which 5 persons A, B, C, D and E
can be seated on round table ifA and D do not sit next (S) 12
to each other
(D) Number of cyphers at the end of the number 50P25

Q.12 Column-I Column-II


(A) Consider a box containing n balls numbered from 1 to n. (P) 12
If 455 is the number of ways to get three balls from the box such that
no two balls are consecutively numbered, then the value of n, is (Q) 17
(B) Let S be the set of points (a, b) in the coordinate plane, where each
of a and b may be – 1, 0, or 1. Number of distinct lines that pass (R) 18
through atleast two points of S, is
(C) All non-empty subsets of {2, 4, 5, 7} are selected. Number of different (S) 20
sums which the elements of these subsets can have, is

BANSAL CLASSES Private Ltd. ‘Bansal Tower’, A-10, Road No.-1, I.P.I.A., Kota-05 Page # 39
PERMUTATION AND COMBINATION

SPECIAL DPP-9

Q.1 Six people are going to sit in a row on a bench. A and B are adjacent, C does not want to sit adjacent
to D. E and F can sit anywhere. Number of ways in which these six people can be seated, is
(A) 200 (B) 144 (C) 120 (D) 56

Q.2 Let m denote the number of ways in which 4 different books are distributed among 10 persons, each
receiving none or one only and let n denote the number of ways of distribution if the books are all alike.
Then :
(A) m = 4n (B) n = 4m (C) m = 24n (D) none

Q.3 There are six periods in each working day of a school. Number of ways in which 5 subjects can be
arranged if each subject is allotted at least one period and no period remains vacant is
(A) 210 (B) 1800 (C) 360 (D) 120

Q.4 The number of all possible selections of one or more questions from 10 given questions, each equestion
having an alternative is :
(A) 310 (B) 210  1 (C) 310  1 (D) 210
Q.5 A team of 8 students goes on an excursion, in two cars, of which one can seat 5 and the other only 4. If
internal arrangement inside the car does not matter then the number of ways in which they can travel, is
(A) 91 (B) 182 (C) 126 (D) 3920

Q.6 The number of ways of choosing a committee of 2 women & 3 men from 5 women & 6 men, if Mr. A
refuses to serve on the committee if Mr. B is a member & Mr. B can only serve, if Miss C is the member
of the committee, is :
(A) 60 (B) 84 (C) 124 (D) none

Q.7 Six persons A, B, C, D, E and F are to be seated at a circular table . The number of ways this can be
done ifAmust have either B or C on his right and B must have either C or D on his right is :
(A) 36 (B) 12 (C) 24 (D) 18

Q.8 There are 2 identical white balls, 3 identical red balls and 4 green balls of different shades. The number
of ways in which they can be arranged in a row so that atleast one ball is separated from the balls of the
same colour, is :
(A) 6 (7 !  4 !) (B) 7 (6 !  4 !) (C) 8 !  5 ! (D) none

Q.9 Product of all the even divisors of N = 1000, is


(A) 32 · 102 (B) 64 · 214 (C) 64 · 1018 (D) 128 · 106

Q.10 One hundred management students who read at least one of the three business magazines are surveyed
to study the readership pattern. It is found that 80 read Business India, 50 read Business world, and 30
read Business Today. Five students read all the three magazines. How manyread exactlytwo magazines?
(A) 50 (B) 10 (C) 95 (D) 25

Q.11 Number of different routes from A to B in the figure if we can travel B


only up or to the right and the intersection point C is closed off, is
(A) 41 (B) 26 (C) 56 (D) 16 C
A

BANSAL CLASSES Private Ltd. ‘Bansal Tower’, A-10, Road No.-1, I.P.I.A., Kota-05 Page # 40
PERMUTATION AND COMBINATION

Q.12 If the number of six digit numbers which contain exactly four different digits such that exactly one digit
repeats, is k (7!), then find k.

Q.13 Column-I Column-II


(A) Number of five digit numbers of the form d1d2d3d4d5 (P) 10C
5
where di's i =1,2,3,4,5 are digits and satisfying d1 < d2  d3 < d4  d5, is
(B) Boby Fischer and Boris Spassky play a unique game series (Q) 11C
4
in a chess tournament. They decide to play on till one of them wins
5 matches. If each match end only in win or Loss. Number of ways
in which series can be won by either of them, is (R) 11C
6
(C) A badminton team has to be selected comprising of 5 students out of
10 students for inter school tournament. Number of ways this can be (S) 2 · 9C5
done if a particular players is to be always included or always excluded
from the team, is

Integer Type Paragraph for question nos. 14 to 16

2 Americal men; 2 British men; 2 Chinese men and one each of Dutch, Egyptial, French and German
persons are to be seated for a round table conference.

Q.14 If the number of ways in which they can be seated if exactly two pairs of persons of same nationality are
together is p(6!), then find p.

Q.15 If the number of ways in which onlyAmerican pair is adjacent is equal to q(6!), then find q.

Q.16 If the number of ways in which no two people of the same nationality are together given by r (6!), find r.

SPECIAL DPP-10

Q.1 There are 100 different books in a shelf. Number of ways in which 3 books can be selected so that no
two of which are neighbours is
(A) 100C3 – 98 (B) 97C3 (C) 96C3 (D) 98C3

Q.2 The sum of all numbers greater than 1000 formed by using digits 1, 3, 5, 7 no digit being repeated in any
number is
(A) 72215 (B) 83911 (C) 106656 (D) 114712

Q.3 Number of positive integral solutions satisfying the equation (x1 + x2 + x3) (y1 + y2) = 77, is
(A) 150 (B) 270 (C) 420 (D) 1024

Q.4 Distinct 3 digit numbers are formed using only the digits 1, 2, 3 and 4 with each digit used at most once
in each number thus formed. The sum of all possible numbers so formed is
(A) 6660 (B) 3330 (C) 2220 (D) none

Q.5 An ice cream parlour has ice creams in eight different varieties . Number of ways of choosing 3 ice
creams taking atleast two ice creams of the same variety, is :
(A) 56 (B) 64 (C) 100 (D) none
(Assume that ice creams of the same variety are identical & available in unlimited supply)
BANSAL CLASSES Private Ltd. ‘Bansal Tower’, A-10, Road No.-1, I.P.I.A., Kota-05 Page # 41
PERMUTATION AND COMBINATION

Q.6 The sum of all the numbers formed from the digits 1, 3, 5, 7, 9 which are smaller than 10,000 if repetion
of digits is not allowed, is
(A) (28011)S (B) (28041)S (C) (28121)S (D) (29152)S
where S = (1+3+5+7+9)

Q.7 The Number of ways in which five different books to be distributed among 3 persons so that each
person gets at least one book, is also equal to the number of ways in which
(A) 5 persons are allotted 3 different residential flats so that and each person is alloted at most one flat
and no two persons are alloted the same flat.
(B) number of parallelograms (some of which may be overlapping) formed by one set of 6 parallel lines
and other set of 5 parallel lines that goes in other direction.
(C) 5 different toys are to be distributed among 3 children, so that each child gets at least one toy.
(D) 3 mathematics professors are assigned five different lecturers to be delivered , so that each professor
gets at least one lecturer.

Q.8 The maximum number of permutations of 2n letters in which there are only a's & b's, taken all at a time
is given by :
2nC 2 6 10 4n  6 4n  2
(A) n (B) . . ...... .
1 2 3 n 1 n

n 1 n  2 n  3 n  4 2n  1 2n 2 n . 1 . 3 . 5 ...... (2 n  3) (2 n  1)
(C) . . . ...... . (D)
1 2 3 4 n1 n n!

Q.9 Number of ways in which 3 numbers in A.P. can be selected from 1, 2, 3, ...... n is :
n  n  2
2
 n  1
(A)   if n is even (B) if n is odd
 2  4
 n 12 n  n  2
(C) if n is odd (D) if n is even
4 4

Q.10 Which of the following statements are correct?


(A) Number of words that can be formed with 6 only of the letters of the word "CENTRIFUGAL" if
each word must contain all the vowels is 3 · 7!
(B) There are 15 balls of which some are white and the rest black. If the number of ways in which the
balls can be arranged in a row, is maximum then the number of white balls must be equal to 7 or 8.
Assume balls of the same colour to be alike.
(C) There are 12 things, 4 alike of one kind, 5 alike and of another kind and the rest are all different. The
total number of combinations is 240.
(D) Number of selections that can be made of 6 letters from the word "COMMITTEE" is 35.

Q.11 The combinatorial coefficient C(n, r) is equal to


(A) number of possible subsets of r members from a set of n distinct members.
(B) number of possible binary messages of length n with exactly r 1's.
(C) number of non decreasing 2-D paths from the lattice point (0, 0) to (r, n).
(D) number of ways of selecting r things out of n different things when a particular thing is always
included plus the number of ways of selecting 'r' things out of n, when a particular thing is always
excluded.

BANSAL CLASSES Private Ltd. ‘Bansal Tower’, A-10, Road No.-1, I.P.I.A., Kota-05 Page # 42
PERMUTATION AND COMBINATION

Q.12 Column-I Column-II


(A) Number of increasing permutations of m symbols are there from the n set (P) nm
numbers {a1, a2, , an} where the order among the numbers is given by
a1 < a2 < a3 <  an–1 < an is
(B) There are m men and n monkeys. Number of ways in which every monkey (Q) mC
n
has a master, if a man can have any number of monkeys
(C) Number of ways in which n red balls and (m – 1) green balls can be arranged (R) nC
m
in a line, so that no two red balls are together, is
(balls of the same colour are alike)
(D) Number of ways in which 'm' different toys can be distributed in 'n' children (S) mn
if every child may receive any number of toys, is

Q.13 Column-I Column-II


(A) Four different movies are running in a town. Ten students go to watch (P) 11
these four movies. The number of ways in which every movie is watched
by atleast one student, is
(Assume each way differs only by number of students watching a movie) (Q) 36
(B) Consider 8 vertices of a regular octagon and its centre. If T denotes the
number of triangles and S denotes the number of straight lines that can
be formed with these 9 points then the value of (T – S) equals
(C) In an examination, 5 children were found to have their mobiles in their (R) 52
pocket. The Invigilator fired them and took their mobiles in his possession.
Towards the end of the test, Invigilator randomly returned their mobiles. The
number of ways in which at most two children did not get their own mobiles is (S) 60
(D) The product of the digits of 3214 is 24. The number of 4 digit natural
numbers such that the product of their digits is 12, is
(E) The number of ways in which a mixed double tennis game can be
arranged from amongst 5 married couple if no husband & wife plays (T) 84
in the same game, is

Q.14 Column-I Column-II


(A) There are 5 balls of different colours and 5 boxes of colours (P) 30
same as those of the balls. The number of ways in which the balls,
one in each box, could be placed such that atleast 2 balls go to the (Q) 31
box of its own colour is
(B) Number of divisors of 1980 that are composite numbers, is (R) 32
(C) Number of seven digit natural numbers in which both 2 and 3 are present
as digits if no two 2's are consecutive in any number, is
(D) There are five different peaches and three different apples. (S) 33
Number of ways they can be divided into two packs of four fruits
if each pack must contain atleast one apple, is
(E) A box contains 6 balls which may be all of different colours
or three each of two colours or two each of three different colours.
The number of ways of selecting 3 balls from the box is
(Assume balls of same colour are alike.)

Q.15 Find the number of ways in which 12 identical coins can be distributed in 6 different purses, if not more
than 3 & not less than 1 coin goes in each purse.

BANSAL CLASSES Private Ltd. ‘Bansal Tower’, A-10, Road No.-1, I.P.I.A., Kota-05 Page # 43
PERMUTATION AND COMBINATION

EXERCISE-2

Q.1(a) How many five digits numbers divisible by 3 can be formed using the digits 0, 1, 2, 3, 4, 7 and 8 if each
digit is to be used atmost once.

(b) Find the number of 4 digit numbers that are multiples of four and can be formed from the digits
0, 1, 2, 3, 4, 5 and 6 such that each digit occurs exactly once.

Q.2 Find the number of 4 digit positive integers if the product of their digits is divisible by 3.

Q.3 There are 2 women participating in a chess tournament. Every participant played 2 games with the other
participants. The number of games that the men played between themselves exceeded by66 as compared
to the number of games that the men played with the women. Find the number of participants & the total
numbers of games played in the tournament.

Q.4 An examination paper consists of 12 questions divided into partsA& B.


Part-Acontains 7 questions & PartB contains 5 questions.Acandidate is required to attempt 8 questions
selecting atleast 3 from each part. In how many maximum ways can the candidate select the questions ?

Q.5 During a draw of lottery, tickets bearing numbers 1, 2, 3,......, 40, 6 tickets are drawn out & then
arranged in the descending order of their numbers. In how many ways, it is possible to have 4th ticket
bearing number 25.

Q.6 Find the number of 6 digit numbers in which repetition of digits are not allowed, even digit appear at
even places, odd digit appear at odd places and the number is divisible by 4.

Q.7 Afirm of CharteredAccountants in Bombay has to send 10 clerks to 5 different companies, two clerks
in each. Two of the companies are in Bombay and the others are outside. Two of the clerks prefer to
work in Bombay while three others prefer to work outside. In how many ways can the assignment be
made if the preferences are to be satisfied.

Q.8 A train going from Cambridge to London stops at nine intermediate stations. 6 persons enter the train
during the journeywith 6 different tickets of the same class. How many different sets of ticket may they
have had?

Q.9 In a University there are 2 Chemists, 3 Physicists and 10 Mathematicians. The President of University
wants to form a scientific committee of 6 members among them. Number of ways in which the committee
can be formed
(a) Without restriction
(b) If Chemist X, Physicist Y and Mathematician Z (i.e. all three) cannot work together.
(c) If atleast half of the committee members must be Mathematicians.
(d) If atmost two Mathematicians can be in the committee.

BANSAL CLASSES Private Ltd. ‘Bansal Tower’, A-10, Road No.-1, I.P.I.A., Kota-05 Page # 44
PERMUTATION AND COMBINATION

Q.10 In how many other ways can the letters of the word MULTIPLE be arranged;
(i) without changing the order of the vowels
(ii) keeping the position of each vowel fixed &
(iii) without changing the relative order/position of vowels & consonants.

Q.11 How many arrangements each consisting of 2 vowels & 2 consonants can be made out of the letters of
the word ‘DEVASTATION’?

Q.12 Each of 3 committees has 1 vacancy which is to be filled from a group of 6 people. Find the number of
ways the 3 vacancies can be filled if ;
(i) Each person can serve on atmost 1 committee.
(ii) There is no restriction on the number of committees on which a person can serve.
(iii) Each person can serve on atmost 2 committees.

Q.13 How many 6 digits odd numbers greater than 60,0000 can be formed from the digits 5, 6, 7, 8, 9, 0 if
(i) repetitions are not allowed (ii) repetitions are allowed.

Q.14 Find the number of ways in which the letters of the word 'KUTKUT' can be arranged so that no two
alike letters are together.

Q.15(a) Find the number of words each consisting of 3 consonants & 3 vowels that can be formed from the
letters of the word “Circumference”. In how many of these c’s will be together.
(b) Find the number of 7 lettered words each consisting of 3 vowels and 4 consonants which can be formed
using the letters of the word "DIFFERENTIATION".

Q.16 How many 4 digit numbers are there which contains not more than 2 different digits?

Q.17 A man has 3 friends. In how many ways he can invite one friend everyday for dinner on 6 successive
nights so that no friend is invited more than 3 times.

Q.18 (i) Prove that : nPr = n1Pr + r. n1Pr1


(ii) If 20Cr+2 = 20C2r3 find 12Cr
(iii) Prove that n1C3 + n1C4 > nC3 if n > 7.
(iv) Find r if 15C3r = 15Cr+3

Q.19 Find the number of five digit numbers that can be formed using the digits 1, 2, 3, 4, 5, 6, 7, 8, 9 in
which one digit appears once and two digits appear twice (e.g. 41174 is one such number but 75355
is not.).

Q.20 Find the number of ways in which 3 distinct numbers can be selected from the set
{31, 32, 33, ....... 3100, 3101} so that they form a G.P.

BANSAL CLASSES Private Ltd. ‘Bansal Tower’, A-10, Road No.-1, I.P.I.A., Kota-05 Page # 45
PERMUTATION AND COMBINATION

Q.21 In an election for the managing committee of a reputed club , the number of candidates contesting
elections exceeds the number of members to be elected by r (r > 0). If a voter can vote in 967 different
ways to elect the managing committee by voting atleast 1 of them & can vote in 55 different ways to elect
(r  1) candidates by voting in the same manner. Find the number of candidates contesting the elections
& the number of candidates losing the elections.

Q.22(a) How many divisors are there of the number x = 21600. Find also the sum of these divisors.
(b) In how many ways the number 7056 can be resolved as a product of 2 factors.
(c) Find the number of ways in which the number 300300 can be split into 2 factors which are relativelyprime.
(d) Find the number of positive integers that are divisors of atleast one of the numbers 1010 ; 157 ; 1811.

Q.23 How many 15 letter arrangements of 5A's, 5 B's and 5 C's have noA's in the first 5 letters, no B's in the
next 5 letters, and no C's in the last 5 letters.

Q.24 Ashop sells 6different flavours of ice-cream. In howmanyways canacustomer choose 4ice-creamcones if
(i) they are all of different flavours (ii) they are non necessarily of different flavours
(iii) they contain only3 different flavours (iv) they contain only 2 or 3 different flavours?

Q.25 Ten dogs encounter 8 biscuits. Dogs do not share biscuits. In how many different ways can the biscuits
be consumed
(a) if we assume that the dogs are distinguishable, but the biscuits are not.
(b) if we assume that both dogs and biscuits are different and any dog can receive any number of biscuits.
(c) if dogs and biscuits are different and every dog can get atmost one biscuit.

Q.26 Determine the number of paths from the origin to the point (9, 9) in the cartesian plane which never pass
through (5, 5) in paths consisting only of steps going 1 unit North and 1 unit East.

Q.27 Find the number of three digits numbers from 100 to 999 inclusive which have any one digit that is the
average of the other two.

Q.28 Let S be the set of first 100 natural number, L is the least common multiple of all the elements in S,
P is the product of all the primes in S and N is the natural number obtained by the ratio L P , then
(i) find the number of cyphers at the end of N.
(ii) find the number of divisors of N which are even but not divisible by 4.
(iii) find the sum of all the odd divisors of N.

Q.29 There are 3 cars of different make available to transport 3 girls and 5 boys on a field trip. Each car can
hold up to 3 children. Find
(a) the number of ways in which they can be accomodated.
(b) the numbers of ways in which they can be accomodated if 2 or 3 girls are assigned to one of the cars.
In both the cases internal arrangement of children inside the car is considered to be immaterial.

Q.30 Find the number of integer between 1 and 10000 with at least one 8 and at least one 9 as digits.

BANSAL CLASSES Private Ltd. ‘Bansal Tower’, A-10, Road No.-1, I.P.I.A., Kota-05 Page # 46
PERMUTATION AND COMBINATION

EXERCISE-3
SECTION-A

(JEE-ADVANCED Previous Year's Questions)

Q.1 Consider all possible permutations of the letters of the word ENDEANOEL
Match the statements / Expression in Column-I with the statements / Expressions in Column-II.

Column-I Column-II

(A) The number of permutations containing the word ENDEA is (P) 5!

(B) The number of permutations in which the letter E occurs in the (Q) 2 × 5!
first and the last position is

(C) The number of permutations in which none of the letters D, L, N (R) 7 × 5!


occurs in the last five positions is

(D) The number of permutations in which the letters A, E, O occurs (S) 21 × 5!


only in odd positions is [JEE 2008, 6]

Q.2 The number of seven digit integers, with sum of the digits equal to 10 and formed by using the digits
1, 2 and 3 only, is
(A) 55 (B) 66 (C) 77 (D) 88 [JEE 2009, 3]

Q.3 Let S = {1, 2, 3, 4}. The total number of unordered pairs of disjoint subsets of S is equal to
(A) 25 (B) 34 (C) 42 (D) 41 [JEE 2010, 5]

Q.4 The total number of ways in which 5 balls of different colours can be distributed among 3 persons so that
each person gets atleast one ball is
(A) 75 (B) 150 (C) 210 (D) 243 [JEE 2012, 3]

Paragraph for Question no. 5 and 6


Let an denote the number of all n-digit positive integers formed by the digits 0, 1 or both such that no
consecutive digits in them are 0. Let bn = the number of such n-digit integers ending with digit 1 and
cn = the number of such n-digit integers ending with digit 0.

Q.5 Which of the following is correct?


(A) a17 = a16 + a15 (B) c17  c16 + c15 (C) b17  b16 + c16 (D) a17 = c17 + b16

Q.6 The value of b6 is


(A) 7 (B) 8 (C) 9 (D) 11 [JEE 2012, 3+3]

Q.7 Let n  2 be an integer. Take n distinct points on a circle and join each pair of points by a line segment.
Colour the line segment joining every pair of adjacent points by blue and the rest by red. If the number
of red and blue line segments are equal, then the value of n is [JEE (Advanced) 2014, 3]

BANSAL CLASSES Private Ltd. ‘Bansal Tower’, A-10, Road No.-1, I.P.I.A., Kota-05 Page # 47
PERMUTATION AND COMBINATION

Q.8 Let n1 < n2 < n3 < n4 < n5 be positive integers such that n1 + n2 + n3 + n4 + n5 = 20.
Then the number of such distinct arrangements (n1, n2, n3, n4, n5) is [JEE (Advanced) 2014, 3]

Q.9 Six cards and six envelopes are numbered 1, 2, 3, 4, 5, 6 and cards are to be placed in envelopes so that
each envelope contains exactly one card and no card is placed in the envelope bearing the same number
and moreover the card numbered 1 is always placed in envelope numbered 2. Then the number of ways
it can be done is
(A) 264 (B) 265 (C) 53 (D) 67
[JEE (Advanced) 2014, 3]

Q.10 Let n be the number of ways in which 5 boys and 5 girls can stand in a queue in such a way that all the
girls stand consecutively in the queue. Let m be the number of ways in which 5 boys and 5 girls can
stand in a queue in such a way that exactly four girls stand consecutively in the queue. Then the value of
m
is [JEE (Advanced) 2015, 4]
n

Q.11 A debate club consists of 6 girls and 4 boys. A team of 4 members is to be selected from this club
including the selection of a captain (from among these 4 members) for the team. If the team has to include
at most one boy, then the number of ways of selecting the team is
(A) 380 (B) 320 (C) 260 (D) 95
[JEE (Advanced) 2016, 3]

Q.12 Let S = {1, 2, 3, ……., 9}. For k = 1, 2, …, 5, let Nk be the number of subsets of S, each containing
five elements out of which exactly k are odd. Then N1 + N2 + N3 + N4 + N5 =
(A) 210 (B) 252 (C) 125 (D) 126
[JEE (Advanced) 2017, 3]

Q.13 Words of length 10 are formed using the letters A, B, C, D, E, F, G, H, I, J. Let x be the number of such
words where no letter is repeated; and let y be the number of such words where exactly one letter is
y
repeated twice and no other letter is repeated. Then, =
9x
[JEE (Advanced) 2017, 3]

Q.14 The number of 5 digit numbers which are divisible by 4, with digits from the set {1, 2, 3, 4, 5} and the
repetition of digits is allowed, is _______. [JEE (Advanced) 2018, 3]

Q.15 Let X be a set with exactly 5 elements and Y be a set with exactly 7 elements. If is the number
of one-one functions from X to Y and is the number of onto functions from Y to X, then the value
1
of ( – ) is______. [JEE (Advanced) 2018, 3]
5!

BANSAL CLASSES Private Ltd. ‘Bansal Tower’, A-10, Road No.-1, I.P.I.A., Kota-05 Page # 48
PERMUTATION AND COMBINATION

Q.16 In a high school, a committee has to be formed from a group of 6 boys M1, M2, M3, M4, M5, M6 and
5 girls G1, G2, G3, G4, G5.
(i) Let 1 be the total number of ways in which the committee can be formed such that the
committee has 5 members, having exactly 3 boys and 2 girls.
(ii) Let 2 be the total number of ways in which the committee can be formed such that the
committee has at least 2 members, and having an equal number of boys and girls.
(iii) Let 3 be the total number of ways in which the committee can be formed such that the
committee has 5 members, at least 2 of them being girls.
(iv) Let 4 be the total number of ways in which the committee can be formed such that the
committee has 4 members, having at least 2 girls and such that both M1 and G1 are NOT in the
committee together.
List-I List-II
(P) The value of 1 is (1) 136
(Q) The value of 2 is (2) 189
(R) The value of 3 is (3) 192
(S) The value of 4 is (4) 200
(5) 381
(6) 461
The correct option is:
(A) P  4; Q  6; R 2; S 1 (B) P  1; Q  4; R 2; S 3
(C) P  4; Q  6; R 5; S 2 (D) P  4; Q  2; R 3; S 1
[JEE (Advanced) 2018, 3]

SECTION-B
(JEE-MAIN Previous Year's Questions)
Q.1 How many different words can be formed by jumbling the letters in the word MISSISSIPPI in which not
two S are adjacent ? [AIEEE 2008]
8
(1) 6. 7. C4 7
(2) 6. 8. C4 6
(3) 7. C4 . C48 6 7
(4) 8. C4 . C4

Q.2 In a shop there are five types of ice-creams available .A child buys six ice-creams.
Statement-1: The number of different ways the child can buy the six ice-creams is 10C5
Statement -2: The number of different ways the child can buy the six ice-creams is equal to the number
of different ways of arranging 6A’s and 4 B’s in a row.
(1) Statement-1 is true, Statement -2 is true; Statement-2 is a correct explanation for Statement-1
(2) Statement-1 is true, Statement -2 is true; Statement-2 is not a correct explanation for Statement-1
(3) Statement-1 is true, Statement -2 is false
(4) Statement-1 is false, Statement-2 is true [AIEEE 2008]

Q.3 From 6 different novels and 3 different dictionaries, 4 novels and 1 dictionary are to be selected and
arranged in a row on a shelf so that the dictionary is always in the middle. Then the number of such
arrangements is
(1) Less than 500
(2) At least 500 but less than 750
(3) At least 750 but less than 1000
(4) At least 1000 [AIEEE 2009]

BANSAL CLASSES Private Ltd. ‘Bansal Tower’, A-10, Road No.-1, I.P.I.A., Kota-05 Page # 49
PERMUTATION AND COMBINATION

Q.4 Statement-1 : The number of ways of distributing 10 identical balls in 4 distinct boxes such that no box
is empty is 9C3.
Statement -2 : The number of ways of choosing any 3 places from 9 different places is 9C3.
(1) Statement-1 is true, Statement-2 is true and Statement-2 is the correct explanation of Statement-1.
(2) Statement-1 is true, Statement-2 is false and Statement-2 is not the correct explanation of Statement-1
(3) Statement-1 is true, Statement-2 is false
(4) Statement-1 is false, Statement-2 is true. [AIEEE 2011]

Q.5 Assuming the balls to be identical except for difference in colours, the number of ways in which one or
more balls can be selected from 10 white, 9 green and 7 black balls is
(1) 630 (2) 879 (3) 880 (4) 629 [AIEEE 2012]

Q.6 Let X = {1, 2, 3, 4 , 5}. The number of different ordered pairs (Y, Z) that can be formed such that
Y  X, Z  X, and Y  Z is empty, is
(1) 25 (2) 53 (3) 52 (4) 35 [AIEEE 2012]

Q.7 Let Tn be the number of all possible triangles formed by joining vertices of an n-sided regular polygon. If
Tn+1 – Tn = 10, then the value of n is
(1) 5 (2) 10 (3) 8 (4) 7 [JEE (Main) 2013]

Q.8 Let A and B be two sets containing 2 elements and 4 elements respectively. The number of subsets of
A × B having 3 or more elements is
(1) 220 (2) 219 (3) 211 (4) 256 [JEE (Main) 2013]

Q.9 The number of points, having both co-ordinates as integers, that lie in the interior of the triangle with
vertices (0, 0), (0, 41) and (41, 0), is
(1) 820 (2) 780 (3) 901 (4) 861 [JEE (Main) 2015]

Q.10 The number of integers greater than 6,000 that can be formed, using the digits 3, 5, 6, 7 and 8 without
repetition, is
(1) 120 (2) 72 (3) 216 (4) 192 [JEE (Main) 2015]

Q.11 If all the words (with or without meaning) having five letters, formed using the letters of the word SMALL
and arranged as in a dictionary; then the position of the word SMALL is
(1) 58th (2) 46th (3) 59th (4) 52nd
[JEE (Main) 2016]

Q.12 A man X has 7 friends, 4 of them are ladies and 3 are men. His wife Y also has 7 friends, 3 of them are
ladies and 4 are men.Assume X andYhave no common friends. Then the total number of ways in which
X and Y together can throw a party inviting 3 ladies and 3 men, so that 3 friends of each of X and Y are
in this party, is
(1) 485 (2) 468 (3) 469 (4) 484
[JEE (Main) 2017]

Q.13 From 6 different novels and 3 different dictionaries, 4 novels and 1dictionaryare to be selected and arranged
in a row on a shelf so that the dictionaryis always in themiddle. The number of such arrangements is
(1) at least 500 but less than 750 (2) at least 750 but less than 1000
(3) at least 1000 (4) less than 500
[JEE (Main) 2018]

BANSAL CLASSES Private Ltd. ‘Bansal Tower’, A-10, Road No.-1, I.P.I.A., Kota-05 Page # 50
PERMUTATION AND COMBINATION

EXERCISE-4
(Potential Problems Based on CBSE)
Q.1 How many 4-letter code can be formed using the first 10 letters of the English alphabet, if no letter can
be repeated ?
Q.2 Given 5 flags of different colours, how many different signals can be generated if each signal requires the
use of 2 flags, one below the other ?
Q.3 Find the number of 4-digit numbers that can be formed using the digits 1, 2, 3, 4, 5 if no digit is repeated.
How many of these will be even ?
Q.4 How many words, with or without meaning can be made from the letters of the word MONDAY,
assuming that no letter is repeated, if
(i) 4 letters are used at a time.
(ii) all letters are used at a time,
(iii) all letters are used but first letter is a vowel ?
Q.5 In how many of the distinct permutations of the letters in MISSISSIPPI do the four I’s not come
together ?
Q.6 In how many ways can the letters of the word PERMUTATIONS be arranged if the
(i) words start with P and end with S,
(ii) vowels are all together,
(iii) there are always 4 letters between P and S ?
Q.7 Find the number of ways of selecting 9 balls from 6 red balls, 5 white balls and 5 blue balls if each
selection consists of 3 balls of each colour.
Q.8 In how many ways can one select a cricket team of eleven from 17 players in which only 5 players can
bowl if each cricket team of 11 must include exactly 4 bowlers ?
Q.9 In how many ways can a student choose a programme of 5 courses if 9 courses are available and 2
specific courses are compulsory for every student ?
Q.10 How many words, with or without meaning, can be formed using all the letters of the word EQUATION
at a time so that the vowels and consonants occur together ?
Q.11 A committee of 7 has to be formed from 9 boys and 4 girls. In how many ways can this be done when
the committee consists of :
(i) exactly 3 girls ? (ii) atleast 3 girls ? (iii) atlmost 3 girls ?
Q.12 How many 6-digit numbers can be formed from the digits 0, 1, 3, 5, 7 and 9 which are divisible by 10
and no digit is repeated ?
Q.13 In an examination, a question paper consists of 12 questions divided into two parts i.e., Part I and Part
II, containing 5 and 7 questions, respectively.Astudent is required to attempt 8 questions in all, selecting
at least 3 from each part. In how many ways can a student select the questions ?
Q.14 Determine the number of 5-card combinations out of a deck of 52 cards if each selection of 5 cards has
exactly one king.
Q.15 From a class of 25 students 10 are to be chosen from an excursion party. There are 3 students who
decide that either all of them will join or none of them will join. In how manyways can the excursion party
be chosen ?
Q.16 In how many ways can the letters of the word ASSASSINATION be arranged so that all the S’s are
together ?

BANSAL CLASSES Private Ltd. ‘Bansal Tower’, A-10, Road No.-1, I.P.I.A., Kota-05 Page # 51
PERMUTATION AND COMBINATION

EXERCISE-5 (Rank Booster)

Q.1 Acrew of an eight oar boat has to be chosen out of 11 men five of whom can row on stroke side only, four
on the bow side only, and the remaining two on either side. How many different selections can be made?

Q.2 In how many ways can a team of 6 horses be selected out of a stud of 16 , so that there shall always be
3 out of A B C A  B  C  , but never AA  , B B  or C C  together.

Q.3 Find the number of permutation of 6 digits from the set {1, 2, 3, 4, 5, 6} where each digit is to be used
exactly once, so that the chosen permutation changes from increasing to decreasing or decreasing to
increasing at most once e.g. the strings like 1 2 3 4 5 6, 6 5 4 3 2 1, 1 2 6 5 4 3 and 6 3 2 1 4 5 are
acceptable but strings like 1 3 2 4 5 6 or 6 5 3 2 4 1 are not.

Q.4(a)Aflight of stairs has 10 steps.Aperson can go up the steps one at a time, two at a time, or any combination
of 1's and 2's. Find the total number of ways in which the person can go up the stairs.
(b) You walk up 12 steps, going up either 1 or 2 steps with each stride. There is a snake on the 8th step, so
you can not step there. Number of ways you can go up.

Q.5 The members of a chess club took part in a round robin competition in which each plays every one else
once. All members scored the same number of points, except four juniors whose total score were 17.5.
How many members were there in the club? Assume that for each win a player scores 1 point , for draw
1/2 point and zero for losing.

Q.6 For each positive integer k, let Sk denote the increasing arithmetic sequence of integers whose first term
is 1 and whose common difference is k. For example, S3 is the sequence 1, 4, 7, 10...... Find the number
of values of k for which Sk contain the term 361.

Q.7 There are n triangles of positive area that have one vertex A(0, 0) and the other two vertices whose
coordinates are drawn independently with replacement from the set {0, 1, 2, 3, 4} e.g. (1, 2),
(0, 1), (2, 2) etc. Find the value of n.

Q.8 How many different ways can 15 Candy bars be distributed between Ram, Shyam, Ghanshyam and
Balram, if Ram can not have more than 5 candy bars and Shyam must have at least two. Assume all
Candy bars to be alike.

Q.9(a) Find the number of non-empty subsets S of {1, 2, 3, 4, 5, 6, 7, 8, 9, 10, 11, 12} such that, no two
consecutive integers belong to S and if, S contains k elements, then S contains no number less than k.
(b) If the number of ordered pairs (S, T) of subsets of {1, 2, 3, 4, 5, 6} are such that S  T contains
exactly three elements is 10, then find the value of .

Q.10 Find the number of three elements sets of positive integers {a, b, c} such that a × b × c = 2310.

BANSAL CLASSES Private Ltd. ‘Bansal Tower’, A-10, Road No.-1, I.P.I.A., Kota-05 Page # 52
PERMUTATION AND COMBINATION

EXERCISE-1

SPECIAL DPP-1

Q.1 (i) 24; (ii) 576; (iii) 360 Q.2 C Q.3 738 Q.4 B
Q.5 (i) 840 ; (ii) 120 ; (iii) 400; (iv) 240 ; (v) 480 (vi) 40; (vii) 60; (vii) 240
Q.6 (i) 120; (ii) 40 ; (iii) 40; (iv) 80; (v) 20 Q.7 47 Q.8 90720
Q.9 A Q.10 4500 Q.11 1024 Q.12 263 Q.13 720
Q.14 1440 Q.15 84 Q.16 999 Q.17 5·49 Q.18 2880
Q.19 72 Q.20 Infinitelymany

SPECIAL DPP-2

Q.1 A Q.2 C Q.3 C Q.4 D Q.5 A


Q.6 A Q.7 C Q.8 154 Q.9 B Q.10 ABC
Q.11 0035

SPECIAL DPP-3

Q.1 967680 Q.2 (a) 60; (b) 107 Q.3 C Q.4 D


Q.5 B Q.6 (a) 213564, (b) 267th Q.7 C Q.8 285
Q.9 32 Q.10 A Q.11 C Q.12 AB

SPECIAL DPP-4
Q.1 1568 Q.2 A Q.3 C Q.4 C
Q.5 64800 Q.6 1736 Q.7 C Q.8 D Q.9 A
Q.10 A Q.11 A

SPECIAL DPP-5

Q.1 25C
5  24C4 Q.2 B Q.3 3150 Q.4 528
Q.5 B Q.6 243 ; 1, 10, 40, 80, 80, 32 Q.7 D
Q.8 378 Q.9 D Q.10 2940

SPECIAL DPP-6

Q.1 8C · 4 ! Q.2 D Q.3 719 Q.4 C Q.5 D


4
Q.6 B Q.7 C Q.8 219 Q.9 D Q.10 B
Q.11 C Q.12 D Q.13 C Q.14 B Q.15 B
Q.16 (A) Q, (B) S; (C) P; (D) R Q.17 212

BANSAL CLASSES Private Ltd. ‘Bansal Tower’, A-10, Road No.-1, I.P.I.A., Kota-05 Page # 53
PERMUTATION AND COMBINATION

SPECIAL DPP-7
Q.1 C Q.2 C Q.3 C Q.4 D Q.5 B
Q.6 256 Q.7 C Q.8 B Q.9 B Q.10 A
Q.11 C Q.12 D Q.13 C Q.14 B Q.15 A
Q.16 6930 Q.17 432 Q.18 10 Q.19 0010

SPECIAL DPP-8

Q.1 D Q.2 A Q.3 B Q.4 C Q.5 A


Q.6 C Q.7 0150 Q.8 A Q.9 8
Q.10 240, 240, 255, 480 Q.11 (A) R; (B) Q; (C) S; (D) P
Q.12 (A) Q ; (B) S ; (C) P
SPECIAL DPP-9

Q.1 B Q.2 C Q.3 B Q.4 C Q.5 C


Q.6 C Q.7 D Q.8 A Q.9 C Q.10 A
Q.11 B Q.12 18 Q.13 (A) R; (B) P, S; (C) P, S Q.14 60
Q.15 64 Q.16 244

SPECIAL DPP-10
Q.1 D Q.2 C Q.3 C Q.4 A Q.5 B
Q.6 B Q.7 BCD Q.8 ABCD Q.9 CD Q.10 ABD
Q.11 ABD Q.12 (A) R; (B) S; (C) Q; (D) P
Q.13 (A) T; (B) R; (C) P; (D) Q; (E) S Q.14 (A) Q; (B) Q; (C) S ; (D) P; (E) Q Q.15 141

EXERCISE-2

Q.1 (a) 744 (b) 208 Q.2 7704 Q.3 13, 156
Q.4 420 Q.5 24C 15C
2· 3 Q.6 1440
Q.7 5400 Q.8 45 C Q.9 (a) 5005; (b) 4785; (c) 4770; (d) 235
6

Q.10 (i) 3359; (ii) 59; (iii) 359 Q.11 1638 Q.12 120, 216, 210
Q.13 240, 15552 Q.14 30 Q.15 (a) 22100, 52, (b) 532770
Q.16 576 Q.17 510 Q.18 (ii) 792 ; (iv) r = 3 Q.19 7560
Q.20 2500 Q.21 10, 3 Q.22 (a) 72; 78120; (b) 23; (c) 32 ; (d) 435
Q.23 2252 Q.24 (i) 15, (ii) 126, (iii) 60, (iv) 105
Q.25 (a) 17C8; (b) 108; (c) 10C8 · 8! Q.26 30980
Q.27 121 Q.28 (i) 1; (ii) 16; (iii) 1920 Q.29 (a) 1680; (b) 1140
Q.30 974

BANSAL CLASSES Private Ltd. ‘Bansal Tower’, A-10, Road No.-1, I.P.I.A., Kota-05 Page # 54
PERMUTATION AND COMBINATION

EXERCISE-3
SECTION-A
Q.1 (A) P; (B) S, (C) Q, (D) Q Q.2 C Q.3 D Q.4 B
Q.5 A Q.6 B Q.7 5 Q.8 7 Q.9 C
Q.10 5 Q.11 A Q.12 D Q.13 5
Q.14 625.00 Q.15 119.00 Q.16 C

SECTION-B

Q.1 3 Q.2 4 Q.3 4 Q.4 1 Q.5 2


Q.6 4 Q.7 1 Q.8 2 Q.9 2 Q.10 4
Q.11 1 Q.12 1 Q.13 3

EXERCISE-4

Q.1 5040 Q.2 20 Q.3 120, 48 Q.4 (i) 360, (ii) 720, (iii) 240
Q.5 33810 Q.6 (i) 1814400, (ii) 2419200, (iii) 25401600
Q.7 2000 Q.8 3960 Q.9 35 Q.10 1440
Q.11 (i) 504; (ii) 588; (iii) 1632 Q.12 120 Q.13 420
Q.14 4C1 × 48C4 Q.15 22C7 + 22C10 Q.16 151200

EXERCISE-5
Q.1 145 Q.2 960 Q.3 62 Q.4 (a) 89, (b) 63
Q.5 27 Q.6 24 Q.7 256 Q.8 440
Q.9 (a) 128; (b) 54 Q.10 40

BANSAL CLASSES Private Ltd. ‘Bansal Tower’, A-10, Road No.-1, I.P.I.A., Kota-05 Page # 55
in Last Nineteen Years

You might also like